Вы находитесь на странице: 1из 103

SKL UJIAN NASIONAL TAHUN PELAJARAN 2017/2018

1
CHAPTER I

INTERPERSONAL AND TRANSACTIONAL COMMUNICATION

1. Congratulations (ucapan selamat) dan Compliment (Memuji)

Untuk memberi selamat, kita dapat mengungkapkan beberapa ungkapan berikut:


Expression Congratulation Response
Congratulations Thank you
Congratulations on … Thank you and the same to you
I’d like to congratulate you. Thank you. I need it.
I’d like to congratulate you on… Thank you very much.
It was great to hear… How nice of you to say so
It was to hear about…. Oh, nothing to it, actually.
Happy birthday to you. Oh, thanks.
Happy new year.
Good luck!
Have a nice holiday

Contoh soal dan pembahasan:


1. Woman: I’d like to congratulate you on your promotion
Man : ...
What’s the best response to the woman’s statement?
A. I dont think so D. How nice of you to say so
B. Others are better E. It’s my pleasure to see you
C. I don’t really want it

Pembahasan: didalam percakapan tersebut seorang perempuan memberikan ucapan


selamat kepada seorang laki – laki atas promosi jabatannya. Ucapan tersebut harus
direspon dengan ucapan terima kasih. Pilihan jawaban yang tepat adalah D. How nice of
you to say so.

Untuk memberikan pujian kita dapat menggungkapkan beberapa ungkapan berikut:

Expression Comliment Response


 You look amazing  Thank you
 You look nice  Thank you very much
 This food is gorgeous  Thanks a lot for your appreciation
 I must give my compliment to the  Thanks a lot for your compliment
winner
 The food is very delicious. You are a  You are so kind to say that
great cook
 What a good answer  You’ve made my day
 You are an excellent student  You’ve given me great encouragement
Contoh soal dan pembahasan:
1. Woman : What a wonderful! You wrote the story amazingly!
Man : ...
Woman : How will the man reply to the woman’s statement?
A. Oh, thanks a lot D. No one questions it
B. I did it seriously E. Everybody loves reading it
C. It is a fantasy story
Pembahasan:
Dialog tersebut merupakan dialog yang belum lengkap. Dialog tersebut harus dilengkapi
dengan respon yang diberikan oleh orang kedua (man). Jawaban atas pujian adalah
terima kasih. Jawaban yang tepat adalah A. Oh, thanks a lot.

2. Expression of Persuading (Membujuk).

Expressions Terjemah
Can’t I persuade you to…? Tidak bisa kah saya membujuk kamu untuk …?
Let’s join… You won’t regret it Mari bergabung … kamu tidak akan menyesal
I’m begging you. Will you…? Aku memohon padamu. Maukah kamu …?

2
Just this once! Sekali ini saja!
How I can persuade you to…? Bagaimana saya bisa membujuk kamu untuk …?
Won’t you… Please? Tidak maukah kamu … Tolonglah?
Why don’t you…? Mengapa kamu tidak …?
Are you really sure you can’t/couldn’t…? Apakah kamu benar-benar yakin kamu tidak bisa /
tidak bisa …?
I really think it would be a pity if we Saya benar-benar berpikir itu akan sangat sayang
didn’t… jika kita tidak …
Are you quite sure you won’t consider…? Apakah Anda yakin Anda tidak akan
mempertimbangkan …?
I really think you’d do well to… Saya benar-benar berpikir Anda akan melakukannya
dengan baik untuk …
Oh, come on! Oh ayolah!
You’re not going to let me down, are you? Anda tidak akan mengecewakan saya, kan?
Not even for me/for my sake? Bahkan bagi saya / demi aku?

3. Expression of Encouraging (Mendorong semangat)

Encouraging is an expression to give someone support, motivation, giving them hope, and
confidence to do something or to make something more likely to exist, happen, or develop,
so he/she can achieve the goals.
Here are some expression to encourage someone:

Expressions Terjemah
Don’t worry. I’m sure you’ll do better the next Jangan khawatir. Saya yakin Anda akan
time melakukan lebih baik waktu berikutnya.
You have my support. Anda memiliki dukungan saya.
You’re doing very well Anda melakukannya dengan sangat baik.
You like it? Go for it! Kamu menyukainya? Lakukanlah!
It wouldn’t hurt Tidak ada salahnya.
Keep at it! Or Keep it up! Tetaplah begitu!
Stick to it! Tetap pada itu!
Give it a shot Cobalah itu
Come on. This is not the end of the world Ayolah. Ini bukan akhir dari dunia.
We feel you should go ahead Kami merasa Anda harus maju
I’m sure you’ll win ! Saya yakin Anda akan menang!
Go on, you can do it! Ayo, Anda bisa melakukannya!
Don’t give up! Jangan menyerah!
No pain, no gain Bersakit-sakit dahulu, bersenang-senang
kemudian
This is for your best Ini adalah untuk kebaikanmu ..
I’m sure you will pass … Saya yakin Anda akan melewati

4. Wish and Hope (Harapan dan Doa)


a. Wish
Wish dapat digunakan untuk mengungkapkan harapan. Contoh:
- I wish (that) I had enough money to buy the house. (Imaginary wish)
- I wish you a happy Idul Fitry. (goodwill)
- I wish to see you at the workshop
b. Hope
Hope dapat dipakai untuk mengungkapkan harapan. Contoh:
- I hope (that) you have a nice Idul Fitri. (goodwill)
- I hope you can come on time. (expected action)

Contoh soal dan pembahasan:


1. Man : Hi Latifa, you look awful. What’s wrong with you?
Woman : A teribble thing happened to me. I invested all of my money in a
new mining company. But now, it’s bankrup
Man : How terrible that is! Is there no way you can get your money back?
I hope you can solve your trouble.
Woman : Yes, I’m tryong so hard to get my money back. I hope the company
can return my money.
Man : I hope so, Latifa.

3
What happened to Latifa?
A. She doesn’t have enough money to invest
B. She wants to invest some money
C. She owns a mining company
D. Her company went bankrupt
E. She lost a lot of money

Pembahasan :
Percakapan tersebut mengenai seorang perempuan yang mengalami masalah dengan
investasinya. Perusahaan dimana dia berinvestasi bankrut. Oleh karena itu jawaban yang
tepat adalah E. She lost a lot of money.

5. Expression of Asking & Giving Opinion (meminta & memberi pendapat)


Asking Opinion Giving opinion
Do you have any thoughts on....? I think (that)….
Do you thin....? In my opinion….
How do you think of Rina’s idea? As I see, …
How do you feel about this decision? If you ask me, I feel…
What is your opinions of the movie? I believe that...
What are your feelings about it? In my point of view
What do you reckon about....? In my opinion...
What is your attitude towards.... My attitude toward....is....

Contoh soal dan pembahasan:


1. Woman : What did you think of the new italian restaurant in the corner of
Margonda Street?
Man : If you like pasta, you must try it there, because the taste is the
best
Woman : How about the price?
Man : You don’t need to worry about it. It suits our pocket

Why did the man think the restaurant is the best choice?
A. Because the chef is the best
B. Becase it is in the Margonda Street
C. Because we can eat Pasta there
D. Because the meal is tasty and cheap
E. Because the restaurant is near the houses

Pembahasan:
Percakapan tersebut mengenai sebuah restaurant italia yang baru diujung jalan
margonda. Menurut pendapat laki-laki tersebut, restaurant tersebut merupakan restaurant
terbagus karena makanannya yang enak dan murah. Karena itu jawaban yang tepat
adalah pilihan E.

6. Expression of Care (Perhatian)

Showing Care Response to Showing Care


To unknown person: Positive:
- Are you hurt? - I’m just fine
- Does it hurt? - Yes, I am okay
- Are you okay? - I’m feeling well
- Are you alright?
To somebody that you know: Negative:
- You seem so tired - No. I’m not.
- How are you feeling? - Nothing, I’m fine
- Do you have toothache? - I broke my arm.
- What’s the matter with you? - I am a little bit tired
- What’s wrong with you? - I’m not feeling good
- You sound you have the flu - Well, I hit the bench
- Are you feeling well today? - Yes, it hurts me very much. I

4
Formal to someone in respect: can’t stand it
- How are you, Sam? - Yes, I’m so tired
- How are you today, Mr. Erwan?
Contoh soal dan pembahasan:
1. Woman : What’s wrong with you Dea?
Man : ......

What’s the best response to the woman’s question?


A. It sounds terrible Daniel
B. Get recovery speedily
C. I have a cough Daniel
D. I’m feeling happy
E. Yes, I’m in a rush

Pembahasan:
Didalam percakapan tersebut, seorang wanita menanyakan keadaan seorang laki-laki
dengan mengatakan, “ada masalah apa Dea?”. Jawaban atas pertanyaan tersebut harus
berhubungan dengan keadaan fisik atau psikologis, jawaban yang terpat adalah C. I have
a cough, Daniel.

7. Expression of Giving Suggestion (Memberikan Saran)

Using Question Why don’t you read the book first?


Why doesn’t he paint his car?
Using How/What about...? How about watching a football match?
What about buying a new computer?
Using modals: Should or could Perhaps you should repair your bike
Maybe you could speak to her.

Contoh soal dan pembahasan:


1. Woman : Hey, John. Why dont you go to the canteen?
Man : No. I don’t have any money in my pocket since my mom
punished me
Woman : What a pity! Perhaps you could borrow me some money now
Man : Thanks a lot, but I’d rather staying here.

What does the woman suggest?


A. Going to the class D. Going to the canteen
B. Staying at the class E. Borrowing some money
C. Having some coffee

Pembahasan:
Didalam percakapan tersebut, perempuan tersebut memberi saran kepada seorang laki –
laki untuk meminjam uang kepadanya karena seorang laki – laki tersebut tidak memiliki
uang. Karena itu jawaban yang terpat adalah pilihan E. Borrowing some money

8. Introduction and Greeting

Introducing your self and other people

Introducing your self Introducing people


 I’d like to introduce myself.  I’d like you to meet … (name)
 My I introduce myself?  This is my friend/boss/etc…(name)
 Let me introduce myself!  Have you met…(name)?
 I want to introduce myself  May I introduce you to …(name/occupation)
 Let me introduce you to ….
 I want to introduce you to ….

1. This is my friend, Jack. Hi Jack. I'm Linda


my brother, Bob.
my friend, Mary Jones.
my boss, Mr. Ritter.
my co-worker, Penny Pitcher.
2. Nice to meet you. Nice to meet you too.

5
Pleased to meet you. Likewise.
Very nice to meet you. And you.
It's a pleasure to meet you.

How to introduce people (in formal situations)

Introducing yourself

I just wanted to introduce myself,


my name is...
I don't believe we've met before,
I'm...
I don't think we've actually met formally yet,

Introducing someone else

I'd like to introduce you to…


There's someone I'd like you to meet, this is…
Have you met…?

Greeting (memberi salam)

Greetings Language in the programme

morning How are you?


sir It's lovely to see you again!
Good madam It's been a long time, hasn't it?
afternoon Mr Jones How are things with you?
Mrs Smith
evening

Examples of situations where you might use formal greetings

Working in the service industry, e.g. a restaurant, hotel, travel agent


Greeting someone older than you
At work, when speaking to your superiors
Meeting a VIP e.g. a politician
Being polite to someone you don't know very well

Expressions Functions
 Good morning/afternoon/evening.  Greeting someone
(formal)
 Hi!/Hello! (informal)
 How are you, Den?  Asking how someone is
 How are you doing
 I’m fi ne, thanks.  Saying how you are
 Very well, thanks.
 Not so bad, thanks.
 See you.  Saying good bye
 Good bye.
 Bye.
 See you soon /later /tomorrow.

Contoh soal dan pembahasan:


1. Man : Hey Lidia, How are you?
Woman : .......

What’s the best response to the man’s question?


A. Not too bad D. I’m happy now
B. I’m fine Tom E. It’s good thank you
C. It is really great

6
Pembahasan:
Didalam percakapan tersebut, seorang laki – laki menanyakan kabar Lidia. Ungkapan
yang tepat adalah adalah pilihan B. I’m fine Tom

9. Inviting (mengundang/mengajak)

Here are some phrases and expressions for inviting in English.


 Do you want to . . .
 Do you wanna . . . (informal)
 Would you like to . . . (more polite)
 How about (V+ing) ?
 How would you like to . . .
 let’s + V1
 Why don’t we …?
 I’d like to invite you to…
 I wonder if you’d like to
Some responds of inviting.
Refusing Receiving
- I’m sorry I can’t - I’d love to
- I’d like to but… - I’d like very much
- I’m afraid I can’t - I’d be happy/glad to accept
- No, let’s not. - Yes, I’d be delighted to.
- That’s good ide

Polite invitations

Checking someone is not busy

Are you free on Friday?


Are you busy on Friday?
What are you doing on Friday?

Would you like...?

Would you like ...a chocolate bar?

...to come to my house for dinner?

I wondered / was wondering

I wondered
...if you'd like to come to my house for dinner
I was wondering

Other expressions

I would very much like it if you could come along


Shall I bring a bottle?

Contoh soal dan pembahasan:


1. Budi : Hi Panjul. Would you like to take part in the workshop this
afternoon?
Panjul : Sure. I’ll be there, Budi.
Budi : Tha’t great. Please come to SMAN 10 Depok building after lunch
Panjul : Alright, thanks.
Budi : My pleasure, Panjul.

What does Budi invite Panjul to?


A. To come to SMAN 10 Depok building D. To come in after lunch
B. To present the workshop E. To join the workshop
C. To attend the meeting

7
Pembahasan:
Didalam percakapan tersebut, bahwa Budi mengundang panjul untuk mengikuti workshop.
Oleh karena itu pilihan jawaban yang tepat adalah pilihan E. To join the workshop

10. Expressing Sympathy (menyatakan rasa simpati)

A. Expressing of sympathy on minor


a. What’s shame
b. What’s pity
c. That’s a nuisance
d. That’s too bad
e. That’s pity
f. Oh dear
B. Expressing of sympathy on serious accident
a. Goodness!
b. How terrible!
c. How Awful!
d. How dreadful!
C. Expressing of sympathy on personal circumstances
a. I’m sorry to hear that
b. I’m sorry about that
c. I’m really sorry for them
d. Please accept my deepest sympathy
e. Send my deepest condolence!
f. Please accept my condolences!

Contoh soal dan pembahasan:


1. Budi : You sound like you have the flu
Panjul : I’ve suffered from the flu since Monday morning
Budi : That’s so bad. I hope you get well soon
Panjul : I hope so. May thanks Budi

What is the conversation about?


A. Panjul is getting well D. Budi shows his sympathy
B. Budi gets the flu E. Panjul hopes to recover
C. Panjul is not feeling good

Pembahasan:
Didalam percakapan tersebut mengenai Panjul yang sedang sakit. Dia menderita Flu.
Untuk itu Budi mengatakan “That’s so bad. I hope you get well soon” untuk menunjukkan
rasa simpatinya. Jawaban yang tepat adalah pilihan D. Budi shows his sympathy

11. Expressing satisfaction and dissatisfaction (kepuasan dan ketidakpuasan)

Ketika kita akan mengungkapkan kepuasan atas kerja seseorang, kita dapat gunakan
ungkapan:
 Well done!
 Great! Good work
 I am satisfied with your work
 You did well
 Your job is satisfactory
 I am so happy about this
 I’m glad to what you’ve done
 It’s really satisfying
Ketika kita akan mengungkapkan ketidakpuasan atas kerja seseorang, kita dapat
gunakan:
 I’m not satisfied with work
 You haven’t done well enough

8
 I am really dissappointed
 Sorry, but your work is not satisfactory
 Oh, no!
 It’s not very nice
 It’s really not good enough

Informal situation
Satisfaction Dissatisfaction
… very pleased with … … displeased with …
… content with … … discontented with …
… satisfi ed with … … dissatisfi ed with …
… very delighted with … … disappointed with …

Formal situation
Satisfaction Dissatisfaction
Super! Horrible!
Great! Very sad!
Terrifi c! Annoying!
Fantastic! Disappointing!
Smashing! Frustrating!

Contoh soal dan pembahasan:


1. Man : Have you seen your score in the report card?
Woman : Yes I have. I am happy with it. I have studied hard for this test.
What about you?
Man : What an awful score! I want to ask Mr. Erwan for correction
Woman : Oh. That’s really bad. I think you should consult him too.

What did the man feel about his test?


A. He go into trouble with Mr. Erwan
B. He asked for a correction
C. He was enjoying the test
D. He was dissatisfied
E. He failed the test

Pembahasan:
Didalam percakapan tersebut laki – laki tersebut mengungkapkan bahwa ia merasa
kurang puas dengan hasil yang ia peroleh dengan mengatakan “what an awful score!”
Oleh karena itu jawaban yang tepat adalah D. He was dissatisfied.

12. Expressing Pleasure, Displeasure (senang & tidak senang)

Pleasure/senang Displeasure/tidak senang


 It’s really delightful/I’m delighted  I’m dissatisfied
 I’m satisfied  We are fed up with…
 That’s great  I feel dosappointed
 That’s wonderful  She is extremely displeased
 It’s really a great pleasure  I’m annoyed
Other expressions
Expression Pleasure Expression Displeasure
a. I’m so happy …. a. I feel …
b. I feel …. b. I’m really sad to …
c. How happy to … c. ….. feel unpleased with ….
d. I’m very pleasure with … d. I feel disappointed.
e. It’s a pleasure to …
f. Pleasure
g. Great!
h. Terrific!
i. I’m pleased.
j. I enjoyed it

9
k. I love it.
l. It was terrifi c.
m. I’m delighted.

Contoh soal dan pembahasan:


1. Woman : Hey Panjul. Thanks for your package. I really like it
Man : You’re welcome. But, I was annoyed with the shipping services.
Woman : Why? I received it safely
Man : Do you know the courier called me many times for your address.
It seems they don’t work proffesionally.

Why does the man feel annoyed


A. The package was received inappropriatley
B. The courier didn’t work professionally
C. The woman spents much money on it
D. The woman ignored the package
E. The courier bothered him

Pembahasan:
Didalam percakapan tersebut laki – laki tersebut mengungkapkan ketidaksenangannya
akan pelayanan yang diberikan karena kurir bekerja tidak profesional. Karena itu jawaban
yang tepat adalah B. The courier didn’t work professionally

13. Expressing Agreement/approval, Disagreement/disapproval (setuju, tidak setuju)

Ketika kita merasa sependapat dengan opini orang lain, kita bisa mengatakan:
Agreement Disagreement
 So do I  Well, I don’t think so
 Yes, I agree with you  I don’t think that is true
 It is certainly  I disagree with …
 Exactly  I wouldn’t say that
 That’s what I want to say  Exactly not
 I am with you  I can’t say so
 I am on your side  On contrary
 Yes, I agree  I don’t buy that idea
 That’s quite true  I’m afraid I entirely disagree
 You’re absolutely right!  I can’t agree
 I’m of exactly the same opinion  I don’t think it’s very good
 I think so  Surely not
 I go along that line  I am sorry, but I have to disagree
 I agree completely  I couldn’t agree less
 That's true.  I’m not sure I can agree
 Absolutely.
 Definitely.
 I couldn't agree more.
 I know what you mean.
 I suppose you’re right

Other expression of disagreeing

Useful vocabulary for disagreeing

No, I don't agree Note: 'no' is usually followed by a statement (see below)
That's not true (quite direct)
I don't accept that (quite formal)

Examples:
No, I don't think that's what happened.

10
No, that's not a good idea.

14. Fear, Anciety (ungkapan ketakutan, kegelisahan)

Fear Respon
I am afraid Don’t be afraid
I am feared There is nothing to be afraid of
I am scared It is nothing
I am terrified
The sound is horrifying
Anciety Respon
I am worried about… Take is easy
I am anxious to know about… Calm down
I wondered if… I know you are worried but…
That made me worried It is not a big deal
I have been thinking about …. Don’t worry
I am afraid if… Stay cool

15. Pain, Relief (ungkapan kesakitan, kelegaan)

Pain Relief
Ouch! I’m very relieved to hear…
That was hurt Finally, it was over
It is painful I feel relieved
It hurts me I feel much better
I’ve got a backache/toothache/stomachache I’m glad it’s over
I feel sore all over That’s a great relief
My eyes hurt I’m extremely glad to hear…
Thank goodness for that
Marvellous
What a relief!

Other expressions
1. Expressions of Pain
• I am suffering from a relapse.
• I feel sick./I feel ill.
• I’m sick.
• Ugh, it’s very painful!
• Oh, it’s killing me!
2. Expressions of Relief
• It’s a relief to know that ....
• Thank God for ....
• I’m glad it was done.
• Thank goodness!
• Thank heavens!
• I’m glad about …!
• It’s a great relief!
• Whew
16. Like/Love & Dislike/Hate (suka/cinta & tidak suka/benci)

Like Dislike
I love it I don’t really like it
I like it I dislike it

11
I am keen on it I am not really interested in…
I am crazy about it I can’t enjoy…
We all enjoy (benda/noun/gerund)…is not my cup of tea
(benda/noun/gerund)…is my cup of tea I can’t stand
I hate it

Language for expressing likes


Subject Adverb Verb Noun

I (really) don't like it


can't stand them
ice cream
Chinese food
playing football
watching TV

About the adverb 'really'.


This adverb as we have seen is very useful in making what you say stronger.
When talking about things you don't like though it can have a different meaning
depending on where you put it in the sentence.
For example:
"I really don't like it!"
This means you have a strong dislike of something.

BUT
"I don't really like it."
This is not very strong. It means that you do not like something, but it is not a very
strong dislike.

Language for expressing likes


Subject Adverb Verb Noun Extra

I (really) like it a lot


love them
ice cream
Chinese food
playing football
watching TV

17. Expressing Embarrasment & Annoyance (Rasa malu dan jengkel)

Embarrassment Annoyance
I am embarrassed I am annoyed
I feel ashamed I had enough with it
Oh my God I can’t bear it any longer
Shame on me You made me annoyed
I don’t feel comfortable You are such a pain in the neck
I feel awkward You made me sick

There are some other expressions you can use to show your annoyance.
Formal Situations Informal Situations

12
I’m extremely displeased with … … really makes me mad.
… is very irritating. I cannot stand …
I’m extremely unhappy about this. Why on earth he didn’t …?

There are some other expressions to show embarrassment, such as:


In Formal Situations In Informal Situations
Formal Situations Informal Situations
What an embarrassment! What a shame!
I must say that it’s an embarrassment. It’s my embarrassment to ...
That’s a real embarrassment. I was so ashamed.

18. Expressing Request (Permintaan)

Request Acceptance Refusal


Would it be possible for you to I should be delighted to I regret to say that we find
Would you be so kind as to come ourselves unable to go
Would you…,please? By all means I’m afraid it’s not possible
Would you mind …? I have no objection I’m afraid not
Any chance of… I’d be happy to Sorry
Can you…? Sure No, I won’t
Yeah Not likely
OK You must be joking
No problem
Mmm

Granting Request
In the dialogue between Ayu and Palupi you fi nd the following expressions:
Ayu : Will you tell me about it?
Palupi : Sure, I will.
Ayu : Let’s try to make lepat sometimes.
Palupi : OK.
Sure, I will and OK are expressions to grant a request.
Here are other expressions that you can use:
- Alright. - Right away
- Certainly. - Of course

19. Expressing Complaint and Blame (Mengeluh dan menyalahkan)

Complaint Blame
I’m not at all satisfied with the service You’re the one to blame
I really do/must objec to the service It’s your fault!
I take great exception to… It’s your mistake!
I want to complain about… You’re wrong
This is crazy! I think you're the only person who could
have done it.
It's your fault for (doing something).

There are a number of formulas used when complaining in English. It's important to
remember that a direct complaint or criticism in English can sound rude or
aggressive. It's best to mention a problem in an indirect manner. Here are some of
the most common:

 I'm sorry to have to say this but...


 I'm sorry to bother you, but...

13
 Maybe you forgot to...
 I think you might have forgotten to...
 Excuse me if I'm out of line, but...

20. Expressing Possibility and Impossibility (kemungkinan dan ketidakmungkinan)

Menyatakan Kemungkinan Menanyakan Kemungkinan


I think there is possibility to … Do you think he/it could…?
I sassume/believe… Would you say we’re capable of…?
In all probability,… Are you capable of…?
it is going to be possible for me to… Are you able to…?
that will probably … Do you have any experience of…?
it’s quite possible … Can you…?
Do you know how to…?
Do you think you can…?

Expressions for Discussing Possibilities


• Would there be any possibility of …?
• Do you think we are capable of …?
• Would it be possible for (somebody) to …?
• I think that would be possible ....
• Is it possible to …?
• Yes, there is a possibility ....

1. Several ways of indicating possibility are:


 It’s possible that he’ll win the game.
 There’s a possibility of his winning the game/ that he will win the game.
 possibly he hasn’t heard the news yet.
 There is a good chance that …..
 There is a little chance that …..
 It is impossible
 Probably She is on the way
 May be he needs more time
 She might not be at home

2. Expressions used to ask possibility or capability of doing something are:


 Would there be any possibility of …..?
 Do you think we are capable of ….?
 Is it possible for me to …?
 Are we capable enough to …?

3. Expressions to show capability are :

 I’m capable of doing it


 I can do it
 There is a chance that I can do it.
 I’m able to do it
 I have the ability to do it.
4. Expressions to show incapability are:

 I can’t do it
 I’m not sure I’m capable of doing it.
 I don’t think I have the ability
 I don’t feel capable of doing it
 I don’t know how to do it.

14
CHAPTER II

LISTENING SECTION

Menentukan gambaran umum atau informasi tertentu/ rinci dari sebuah percakapan
interpersonal / transaksional secara formal atau informal

Menentukan respon yang tepat terhadap percakapan transaksional/ interpersonal secara


formal atau informal

A. SHORT DIALOGUE

Dua indicator tersebut, pertanyaan yang sering muncul sebagai berikut:

Menentukan gambaran umum 1. What is the conversation about?


2. What does the man/woman imply?
3. What does the man/woman say about?
4. What can we infer from the conversation?
Menemukan informasi tertentu 5. What would the man/woman probably do?
6. What does the man/woman say about?
7. Where does the conversation take place?
8. Who is the woman/man?

Contoh soal dan pembahasan:

Woman : Holiday is coming up, what’s your plan?


Man : I’m going to Bali. I will go to some tourist resources such as: Bedugul,
Kintamani, Kuta, and Kuta square. I will go shopping, swimming and see beautiful scenery.
I will be there for a week.
Woman : Oh it sound wonderful. I envy you.

What is the main topic of the conversation?


1. A. Having a journey for a week.
B. Going shopping.
C. Going swimming.
D. Beautiful scenery.
E. Discussing holiday plans.

Pembahasan: Didalam percakapan tersebut wanita (woman) menanyakan rencana


liburan si pria (man). Dan dalam liburannya si pria akan selama satu minggu. Jadi jawaban
yang tepat adalah A.

Woman : Hello Jack, where are you now?


Man : I’m still at the office, why?
Woman: I need to see you. Keep wait for me there.

Where will the woman go?


2. A. She will go to school.
B. She will go to her factory.
C. She will go to the hospital.
D. She will go to the man’s office.
E. She will go to a department store.

Pembahasan: Dari percakapan kita tahu bahwa siwanita mencari si pria. Dan dia bilang
untuk tunggu di kantor. Oleh karena itu, jawaban yang tepat adalah D.

15
Man : When will you come over for dinner. Will you come tonight?
Woman : Not tonight. I promise to go to concert with my brother.
Man : Well, how is about Friday then?
Woman : That sounds great

Why is the woman not able to come to dinner?


3. A. She has broken her own promises.
B. She cannot cook the dinner well.
C. She is going to a concert.
D. She practices some sounds.
E. She will perform in the concert.

Pembahasan: dari percakapan bisa diketahui bahwa si wanita tidak bisa datang untuk
dinner karena harus menemani kakaknya pergi ke konser. Oleh karena itu jawaban yang
tepat adalah C.

Woman : Look at me. Does this long red dress fit me John?
Man : I don’t think so. I think the green long dress is better than the red one.
Woman : How about the short black one then?
Man : Yes, that suits you. You look gorgeous with that one.

Which dress fits the woman?


4. A. The short red one.
B. The short green one.
C. The short black one.
D. The long red one.
E. The long green one.

Pembahasan: dari percakapan diatas, si pria memberikan saran baju yang cocok adalah
yang berwarna hitam. Oleh karena itu jawaban yang tepat adalah C.

Woman : Dani, come here, please. I want to talk to you


Man : Yes Mam. What has been wrong?
Woman : Mr. Rudy told me that you were late to his class. Why?
Man : I’m sorry mam. I missed my book.
Woman : Now tell me you won’t do it again.
Man : OK, Mam. I promise I will never do it again

What can we infer from the conversation?

5. A. The teacher punished the boy in the class


B. The boy didn’t attend Mr. Rudy’s class
C. The boy promised not to be late
D. The boy’s book has lost in the class.
E. Mr. Rudy’s class was dismissed.

Pembahasan: Pertanyaan diatas menanyakan apa yang dapat kita simpulkan dari
percakapan. Di dalam percakapan dinyatakan bahwa laki – laki tersebut berjanji tidak
akan pernah melakukannya lagi, “OK Mam, I promise I will never do it again”. Jawaban
yang benar adalah C.

16
Latihan soal 1

Man : Hot today, isn’t it?

Woman : Yes, it is. I wish that it would rain and cool off.

Man : Me, too. This is unusual for March. I don’t remember it’s ever been so hot and
dry in March before.

Q: According to the conversation, what kind of weather is usual for March?


A. Warm
B. Drier
C. Hotter
D. Cooler
E. Very Hot

Latihan soal 2

Man : An oil and gas company is carrying out a science competition to support its
effort to provide means of educating the nation youths.

Woman : Are all students allowed to take part in the competition?

Man : Oh Yes. College students from all the country’s provinces

Q: What’s the topic of the dialogue?


A. Sport competition
B. Nation’s youth education
C. The country’s college students
D. Oil and gas company’s science competition
E. Means of educating the nation’s youth

Latihan soal 3

Man : I don’t know what to order. I could drink everything on the menu

Woman : Why don’t you try guava juice, orange juice or ice tea

Man : Guava juice sounds good. I’ll take it

Woman : I think I’ll have a glass of cola float

Q: What will the woman do?

A. Take a guava juice


B. Order an orange juice
C. Try an ice tea
D. Order a cola float
E. Drink mineral water

Latihan soal 4

Man : Who wrote that exciting spy adventure novel “topaz’?

Woman : That wa Leon Uris

Man : Didn’t he also write those famous stories about bullfighting in Pamplona
Spain?

Woman : No, that’s Ernest Hemingway

Q : What did Leon Uris do?

A. He was a spy
B. He was a bullfighter

17
C. He wrote famous stories
D. He wrote about bullfighting
E. He wrote an adventure novel

Latihan soal 5

Woman : Why don’t you take the bus to work? Driving in rush hour traffic is terrible

Man : I would. But I’m affraid there won’t be any seat left by the time the bus gets to
my stop.

Q: Why does the man not want to get on the bus?


A. The man takes the bus to work
B. The man wants the woman to go with him
C. The woman prefers driving in the rush hour
D. The woman wants to ride to work with the man

LATIHAN SOAL UN TAHUN 2017


NO SOAL INDIKATOR SOAL

1 What does the woman express? Diperdengarkan sebuah percakapan sederhana


tentang mengucapkan dan merespon ucapan
A. Regret selamat bersayap (± 4 (empat) pertukaran peran),
B. Congratulation peserta didik dapat menentukan jawaban atas
C. Sympathy
pertanyaan tentang informasi tertentu dari
D. Expectation
percakapan (5 pilihan jawaban).
E. Hope
2 What is the best solution they have? Diperdengarkan sebuah percakapan sederhana
tentang menyatakan harapan dan doa (± 4
A. Make themselves crazy pertukaran peran), peserta didik dapat
B. Come late to school
menentukan jawaban atas pertanyaan tentang
C. Call the teacher
informasi rinci dari percakapan (5 pilihan
D. Hope and pray
E. Miss the first lesson jawaban).

3 What are they talking about? Diperdengarkan sebuah percakapan sederhana


tentang menyatakan dan menanyakan tentang
A. Something wrong pendapat dan pikiran (± 4 pertukaran peran),
B. A terrible cough
peserta didik dapat menentukan jawaban atas
C. A problem to sleep
pertanyaan tentang gambaran umum isi
D. How to stop smoking
E. The effect of smoking percakapan (5 pilihan jawaban).

4 Why does the woman feel happy? Diperdengarkan sebuah percakapan sederhana
tentang ungkapan mengawali penyampaian berita
A. She has met her friend atau informasi yang mengejutkan serta responnya
B. She has been on the news
(± 4 pertukaran peran), peserta didik dapat
C. She has been accepted at
menentukan jawaban atas pertanyaan tentang
medical faculty
D. She has become a doctor informasi rinci dari percakapan (5 pilihan
E. She deserves to study hard jawaban).

18
B. QUESTIONS – RESPONSES

Selanjutnya dalam indicator ini juga akan ada beberapa pertanyaan yang
mengharuskan kita untuk melengkapi respon yang tepat terhadap sebuah dialog yang
tidak komplit. Soal dan pilihan jawaban akan diperdengarkan. Kita hanya akan
mendapatkan perintah Mark your answer on your anwer sheet.
Berbeda dengan soal listening sebelumnya, pada bagian ini siswa tidak dapat
kesempatan untuk memprediksi soal apa yang akan muncul. Oleh karena itu, kemampuan
siswa untuk mendengarkan percakapan yang diperdengarkan sangat penting.
Tips.
 Perhatikan dengan seksama pembicara yang terakhir karena pembicara yang
terakhir menyampaian ungkapan yang menentukan jawabannya
 Simaklah pilihan jawaban dengan sebaik – baiknya

Simak kembali materi tentang ekspresi – ekspresi dalam percakapan transaksional atau
interpersonal.

Contoh soal dan pembahasan:

1. Man: do you have any problem with our service in this resort mam?
What is the most appropriate respond?
A. Sorry I cannot hear it
B. Thank you very much
C. No, I’m satisfied with it ….
D. Yes, please send it to.

Pembahasan: Respon yang tepat terhadap kalimat tersebut menggunakan ekspressi


puas. Jawaban yang tepat adalah C. No, I’m satisfied with it.

2. Man : The air pollution caused by cars and factories create the greenhouse
effect and destroy the land.
What is the man most likely to reply?
A. It’s good for our health
B. Everything is fine thank you
C. Yes, this is a very bad situation….
D. It would be delighted
Pembahasan: Ekspresi yang tepat adalah ekspresi setuju terhadap pendapat. Jawaban
yang benar adalah C. Yes, this is a very bad situation.

3. Man : What’s the matter Darla? You look sad today.


Woman : My father is very ill. And has been hospitalized since yesterday
What’s the appropriate respond?
a. I hope he would get better soon
b. I think you should take the rest
c. I’m glad to hear that
d. I suggest you cheer up
Pembahasan: Ekspresi yang tepat adalah ungkapan simpati. Yaitu jawaban a. I hope he
would get better soon.

19
4. Man : Hi, Reta. I’m having dinner party this weekend. I would be grateful if you
could join us.
Woman : …..
What’s the appropriate respond?
a. I’m so happy to hear it
b. See you this weekend
c. I’m fine thank you
d. I’m definitely Okay
e. Sure, I’d love to
Pembahasan: Ekspresi yang tepat adalah ungkapan menerima undangan. Yaitu jawaban
e. Sure, I’d love to.

Latihan soal 1
Woman : Whose car is it in front of my house?
Man : It’s yours, madam. Congratulations! Our company has decided that
you won the painting competition which was held last month.
Q: What would the woman reply? Mark your answer on your anwer sheet.
A. You’d better not do that
B. The painting competition was really tough
C. Are you sure? I can’t believe it
D. The car is very unique

Latihan soal 2
Man : Hi, Anita. We’ve finished the Natinonal Exams today. We can
refresh our mind. How did you feel?
Q: What would the woman probably respond? Mark your answer on your anwer sheet.
A. I’m very doubtful
B. I’m very stressed
C. I’m very upset
D. I’m very relieved

Latihan soal 3
Man : I’m sorry you didn’t pass your cast for your new movie.
Woman : Yeah, the director of the movie told me tht I wasn’t suitable for the
character of being a poor lady
Q: What would the man reply to express encouragement?
Mark your answer on your anwer sheet.
A. It’s alright. The movie is too expensive
B. Are you okay? You must hate the movie
C. Don’t worry. You still have many other opportunities.
D. It’s really a shameful thing for the next movie.

20
LATIHAN SOAL UN 2017
5 Mark your answer on your answer sheet. Diperdengarkan sebuah percakapan
sederhana dengan ungkapan
What is the best response for Donna?
meminta perhatian bersayap
A. Yes sir, That’s right. (extended) yang tidak lengkap serta
B. I'm sorry sir, I promise 4 pilihan jawaban, peserta didik
C. I don’t care sir, I don’t like it. menentukan pilihan yang paling
D. I'm sorry sir I will study better tepat untuk melengkapi responnya.
6 Mark your answer on your answer sheet. Diperdengarkan sebuah percakapan
sederhana dengan ungkapan saran
What is the best response for the father?
dan tawaran yang tidak lengkap
A. You must go to the park for the test serta 4 pilihan jawaban, peserta
today. didik dapat menentukan pilihan yang
B. Take a rest and I'll call your paling tepat untuk melengkapi
teacher. responnya.
C. It’s up to you and I don't care.
D. I will take the test.

7 Mark your answer on your answer sheet. Diperdengarkan sebuah percakapan


sederhana dengan ungkapan
What does the man probably reply?
menawarkan jasa yang tidak
A. sorry I don't understand you lengkap serta 4 pilihan jawaban,
B. it's okay don't disturb me peserta didik dapat menentukan
C. you know actually I'm busy pilihan yang paling tepat untuk
D. you’re welcome melengkapi responnya.

Menentukan gambar yang tepat sesuai dengan informasi yang ada didalam percakapan
interpersonal/ transaksional secara formal atau informal

Menentukan gambar yang tepat sesuai dengan teks monolog yang diperdengarkan

C. PICTURE IDENTIFICATION

Pada bagian ini siswa akan diuji kemampuannya dalam mencerna isi dari suatu
perckapan dan monolog singkat. Kemampuan tersebut diuji dengan mengidentifikasi
gambar yang sesuai dengan apa yan didengar. Siswa akan dihadapkan dengan 5 pilihan
gambar.

Pertanyaan yang sering muncul berkaitan mengenai gambaran umum percakapan


atau informasi mendetail dalam percakapan sebagai berikut:

 Which picture goes with the dialogue?

 Which picture matches the man’s favorite interest?

 What tools should be used for checking the tyres?

 Where does the conversation take place?

 Where would the man/woman go?

 What would the man/woman do?

 What will the man and the woman do?

Percakapan yang didengar akan berhubungan dengan ekspresi – ekspresi dalam


percakapan transaksional atau interpersonal. Sedangkan untuk soal monolog akan
berhubungan dengan teks – teks.

21
Contoh soal dan pembahasan

Man : Hi Jessy. I didn’t see you yesterday. What happened to you?

Woman : I had a cold and fever yesterday, so I had to take a rest.

Man : Are you feeling better now?

Woman : Not yet, I’m still feeling under the weather

Man : What can I do for you?

Woman : Nothing thanks. I bought some medicine yesterday

Man : Hope you will get better soon.

Woman : Thanks

A B C
. . .

D E
. .

Pembahasan: didalam percakapan, seorang perempuan menyatakan bahwa dia tidak


hadir karena mengalami flu dan demam. Oleh karena itu jawaban yang tepat adalah D.

2. Man : I think I’ll take a kilo of it. Juli, Do you want any apple, mango or banana?

Woman : I need pineapple for my diet.

What will the woman take?

A B C
. . .

D E
. .

Pembahasan: Perempuan tersebut diakhir dialogue mengatakan bahwa dia mau


pineapple. Sehingga jawaban yang tepat adalah. A.

22
3. Do you know what seaweed is? Seaweed is the common name for countless species of
marine plants and algae that grow in the oceans as well as in rivers, lakes and other water
bodies. Some seaweeds are microscopic, such as the phytoplankton that live suspended
in the water column and provide the base for most marine food chains.

Question: Where can the seaweed grow?

B D

C E

Pembahasan: Dari teks monolog diatas disebutkan bahwa seaweed grows in the oceans.
Jawaban yang paling tepat adalah gambar D.

Menentukan gambaran umum atau informasi tertentu/tersirat/rinci dari sebuah teks


monolog yang diperdengarkan

Latihan soal 1

Woman : What is your favorite music Ben?


Man : Western music, but I like popular and classical music
Woman : Can you play the guitar?
Man : A little bit. I also like to play violin, piano, percusison and kulintang
Woman : Oh really? That’s amazing, then. Which instrument are you most
interested in?
Man : The piano. It’s more flexible
Woman : That’s marvelous
Q: Which picture matches the boy’s favorite interest?
D

23
Latihan soal 2

Woman : I think I gain weight. I’ve eaten a lot recently.


Man : Why don’t you cheat your weights? I put out scales next to the
bathroom
Q: Which picture fits the conversation?

A D

Masih dalam soal Picture Identification, dalam variasi kedua juga biasanya disodorkan
gambar – gambar kemudian kita harus mendengarkan monolog singkat. Setelah itu akan
ditanyakan gambar mana yang sesuai dengan yang ditanyakan.

Latihan soal:

Dikaset diperdengarkan:

Woman : Termites are groups of social insects. They are commonly known
especiall in Australia as white ants. They divide labour among
castes producing overlapping generation and collectively takiing
care of the young. They live in colonies. People considered termites
as pest, as they can cause serious structural damage to buildings,
crops or forest plantations.

Q: Which picture fits the conversation?

A D

B E

24
Latihan Soal

Dikaset diperdengarkan:

Man : A car is means of transportation. Almost everybody goes to work by a car.


Therefore, a car is very crucial. It needs to be service by the owner
regularly. Beside servicing the engine, the owner should pay attention to all
the tyre. Inside the car, there should be important tools such as scissors lift
jack, or tyre pressure gauge. It’s very important to change the tyre when it’s
flat. A car doesn’t need to have a night enginge generator, a tryre
compressor, or a tyre changer, digital tyre inflator but all the should be
checked regulalry before driving.

Q: What tool should be used for checking tyre?

A B

C
D

LATIHAN SOAL UN 2017

8 Which is the place the speaker describes? Diperdengarkan sebuah


monolog sederhana yang
lengkap tentang bangunan
bersejarah terkenal
(misalnya benteng,
museum, tempat ibadah,
peninggalan sejarah),
peserta didik dapat
menentukan gambar yang
tepat sesuai dengan isi
monolog (5 pilihan
jawaban).

25
9 Which picture is described in the monologue? Diperdengarkan sebuah
monolog sederhana yang
lengkap tentang tempat
wisata, peserta didik dapat
menentukan gambar yang
tepat sesuai dengan isi
monolog (5 pilihan
jawaban).

10 Which picture is described in the monologue? Diperdengarkan sebuah


monolog sederhana yang
lengkap tentang benda
(misalnya barang-barang
antik dan peninggalan
sejarah seperti artifak dan
lain-lain), peserta didik
dapat menentukan gambar
yang tepat sesuai dengan
isi monolog (5 pilihan
jawaban).

11 Which picture is described in the monologue? Diperdengarkan sebuah


monolog sederhana yang
lengkap tentang gejala
dan peristiwa alam,
peserta didik dapat
menentukan gambar yang
tepat sesuai dengan isi
monolog (5 pilihan
jawaban).

26
D. MONOLOGUE

Pada bagian listening yang terakhir, siswa akan diuji kemampuannya dalam mencerna
monolog. Monolog adalah penjabaran secara lisan yang disampaikan oleh satu orang.
Didalam UN, teks monolog yang akan diujikan adalah recount (biography), news item,
narrative, descriptive, analytical exposition, discussion dan lainnya.

Pertanyaan yang sering muncul sebagai berikut:

Indikator Pertanyaan

Gambaran Umum  What is the monolog about?


 What is the monolog talking about?
 What is the topic of the monolog?
Menemukan Informasi Rinci  Where did the accident happen?
 What did the man buy?
 Who designed the building?
 Etc….
Menemukan Informasi  Who is the speaker probably?
Tersirat  Where the monolog is probably heard?
 What will the man/woman probably do
next?

Contoh soal dan pembahasan:

Descriptive

Man: I have a pet. It’s a cat. I call her Lolie. Lolie is an American breed. She is
small, fluffy and cute. She has thick, brown fur. When I cuddle her, the fur
feels soft.

Narrator: What is the man talking about?

a. A cat d. A cute bird

b. A dog e. A soft rabbit

c. A fish

Pembahasan: Monolog diatas menceritakan tentang seseorang yang mempunyai kucing.


Jawaban yang benar adalah A.

News item

Man: Good Morning, listeners. Welcome back to NG radio with news line. Recently,
there are four schools in this district which have been infected by typhoid
bacteria or have displayed symptoms of it. The infected schools are facing
problems with cleanliness and sanitation. Many students complained that
sanitation is not an important matter in those schools. The schools are closed
for two days as a precautionary effort from the government and medical
institution in the district.

Narrator: What is the monolog talking about?

a. The danger of typhoid bacteria d. students’ complaint

b. Virus spread along the school e. school regulation

c. Protection from typhoid bacteria

27
Pembahasan: Dalam monolog diatas diceritakan bahwa radio NG menyiarkan berita
mengenai 4 sekolah di wilayah kabupaten yang terjangkit bakteri Tifus. Oleh karena itu
jawaban yang tepat adalah A.

Latihan Soal
Angkor Wat (Khmer: or "Capital Temple") is a temple complex
in Cambodia and the largest religious monument in the world, on a site measuring 162.6
hectares (1,626,000 m2; 402 acres).[1] It was originally constructed as a Hindu temple of
god Vishnu for the Khmer Empire, gradually transforming into a Buddhist temple towards
the end of the 12th century.[2] It was built by the KhmerKing Suryavarman II in the early
12th century in Yaśodharapura. Angkor Wat combines two basic plans of Khmer temple
architecture: the temple-mountain and the later galleried temple. It is designed to
represent Mount Meru, home of the devas in Hindu mythology: within a moat and an outer
wall 3.6 kilometres (2.2 mi) long are three rectangular galleries, each raised above the
next. At the centre of the temple stands a quincunx of towers. Unlike most Angkorian
temples, Angkor Wat is oriented to the west; scholars are divided as to the significance of
this. The temple is admired for the grandeur and harmony of the architecture, its
extensive bas-reliefs, and for the numerous devatas adorning its walls.

Q: What is the monologue about?


A. Gallery Complex
B. Buddhist mythology
C. Cambodia
D. Khmer Empire
E. Angkor Wat

LATIHAN SOAL UN 2017

12 What is the topic of the monologue? Diperdengarkan sebuah monolog


pembacaan berita sederhana dari radio
A. West Nusa Tenggara (NTB)
tentang kejadian/peristiwa terkini,
B. Hosting a festival in NTB
peserta didik dapat menentukan pilihan
C. Coral Reefs
D. The conservation destinations jawaban yang tepat atas pertanyaan
E. The environmental issues. tentang topik berita (gambaran umum) isi
monolog.

13 What will happen if the festival is Diperdengarkan sebuah pembacaan


successful? berita sederhana dari radio tentang
kejadian/peristiwa terkini, , peserta didik
A. More hotels and restaurants
dapat menentukan jawaban yang tepat
will be free.
atas pertanyaan tentang informasi
B. More festivals will be
conducted by the tourists. tersirat dalam monolog.
C. More income will be generated
for the government and local
people
D. More islands will be explored
for hotels and restaurants
E. More trees will be removed for
the islands
14 What is the monologue about? Diperdengarkan sebuah monolog
biografi pendek tentang tokoh terkenal,
A. A biography of General
peserta didik dapat menentukan jawaban
Soedirman
yang tepat atas pertanyaan tentang

28
B. The family of General gambaran umum isi monolog (5 pilihan
Soedirman jawaban).
C. The death of General
Soedirman
D. A spirit of General Soedirman
for the Indonesian Armed
Forces
E. The military forces
commanded by General
Soedirman
15 What can we infer from the Diperdengarkan sebuah monolog
monologue? biografi pendek tentang tokoh terkenal,
peserta didik dapat menentukan pilihan
A. His uncle’s name was also
jawaban yang tepat atas pertanyaan
Soedirman
tentang informasi tersirat dalam monolog
B. January is the month of
Maulud (5 pilihan jawaban).
C. Soedirman was shot and died
in the military war
D. Soedirman died when he was
relatively young
E. Soedirman died on 1 March
1949

29
CHAPTER III

READING SECTION

Menentukan gambaran umum atau informasi rinci/tersirat/tertentu atau makna


kata/frasa/kalimat atau pikiran utama paragraph/rujukan kata atau tujuan komunikasi dari
teks berbentuk letter/email.

Dibawah ini adalah macam-macam bentuk pertanyaan yang biasa ada dalam soal
Reading:
a) Menemukan gambaran umum/topic isi bacaan/teks.
Contoh pertanyaan :
∞ Which of the following is the most suitable title…?
∞ What is the suitable topic of the passage?
∞ The text mainly tells us about____.
b) Menemukan informasi tertentu/khusus dari bacaan.
Contoh pertanyaan :
∞ When did she make her first solo flight? In…
c) Menemukan informasi rinci yang tersurat dari bacaan teks/dialog.
Contoh :
∞ Which of the following requirements is not mentioned in the passage?
∞ “They may be classified in several different ways…”
∞ The underlined word refers to ….
d) Menemukan informasi rinci yang tersirat dari bacaan teks/dialog.
Contoh :
∞ Which statement is NOT TRUE according to the text?
∞ The following are TRUE about Maria EXCEPT…
e) Menentukan main idea yang tersurat/tersirat dari suatu paragrap.
Contoh :
∞ What is the main idea of the passage?
∞ The fourth paragraph tells us ____.

f) Menentukan makna kata, frasa dan kalimat berdasarkan konteks.


Contoh :
“Brownie is Chinese breed, it is small, fluffy. And cute.”
The underlined word mean ____

g) Menentukan type text yang digunakan penulis.


Contoh :
What type text is used by the writer?
The text above is in the form of _____.

h) Menentukan communicative purpose/tujuan kominikativ sebuah teks


Contoh :
The communicative putpose of the text above is ___.
The purpose of the text is _____
.
i) Menyusun kalimat dengan baik.
Contoh :
The best order of the sentences above is …
The best arrangement of the sentences to make a good paragraph is …

30
A. SHORT FUNCTIONAL TEXTS

1. PERSONAL LETTER

Personal letter dapat berupa surat pos dan surat elektronik (e-mail). Surat pribadi
digunakan untuk beberapa tujuan tertentu yaitu untuk memberi salam, memberikan
ucapan selamat, menceritakan pengalaman masa lalu, memberikan pendapat, meminta
sesuatu, dll.

Struktur dari personal letter adalah sebagai berikut:

 The heading includes the date of the letter, the sender address.

 The opening of the letter consists of the salutation and the recipient

 The body contains the information that would be delivered to the recipient

 The closing contains the complimentary closing and the writer’s signature

Example of personal letter.


Your
Retno
1207 Pine Street address
Tasikmalaya
Receiver’s name 11 Oktober 2016 The date
and address Riyani
Jln. Cendana. No. 16

The Greeting Dear Riyani


Hello my friend, how are you doing, there? I hope you are in good condition.
I am sending this letter to give you some good news.
It’s been one year we separated, since you moved to Makassar. I miss you
so much. I’m really looking forward to meeting with you soon. After waiting
for so long, i finally granted permission to visit you in Makassar. Maybe next
month I will go there by plane. When I got there, you have to take me
around to the tourist attractions there. I really wanted to visit the Losari
beach. See you there buddy.
Complementary
salutation With love, Signature and
sender’s name
Retno

Greeting

Ada beberapa cara untuk melakukan greeting/ opening salutation pada personal
letter:

 Dear (yang paling umum digunakan)

 Good day atau Hi/Hello (netral dan kurang formal)

Greeting atau salutation biasanya juga diikuti dengan recipient name (nama
penerima). Bisa juga ditambahkan title seperti berikut:

Mr : for male Miss :for unmarried female

Mrs : for a married female Ms : for female whose status is unknown

Complimentary closing

Ada beberapa cara juga untuk menulis complimentary closing:

- Regards (neutral – there is no close relation)

- Sincerely yours, cordially (warmer relationship between the sender and the
recipient)

- With love, (close relation)

31
Contoh soal dan pembahasan

Mr. David Beckham


Blitz Computer, Ltd

Dear Mr. Beckham,


I appreciate your prompt service. The computer I ordered arrived in time. I have
checked the whole package. All the packages are in good condition and the computers are
really excellent.
The computers are of high standard that enable us to process data rapidly. In
addition, the software is very practical and updated regularly.
Thank you once again for your contribution.
Sincerely,

Kanjeng Wahid
Manager PT. Angin Gede, L.Td

1. Why was Mr. Kanjeng Wahid satisfied with the computers?


A. They enabled him to process data quickly
B. Their price is reasonable
C. They are very efficient
D. They are compatible
E. They are compact

Pembahasan: Didalam surat disebutkan bahwa computer uang dikirim merupakan


computer dengan kualitas bagus yang mempercepat proses mengolah data. Dengan
demikian, dapat dikatakan bahwa pak Kanjeng puas dengan computer tersebut. Jawaban
yang tepat adalah pilihan: A

2. What is the purpose of the letter?


A. To order new computers
B. To request computer repairs
C. To express thanks for the computer supplies
D. To complain about the quality of the computers
E. To inquiry about the installation of the computers

Pembahasan: Pada paragraph pertama surat tersebut, disebutkan bahwa Pak Kanjeng
Wahid memberikan apresiasi terhadap layanan yang diberikan oleh Blitz Computer, L.Td.
Oleh karena itu, tujuan dari surat ini adalah untuk mengucapkan terima kasih. Jawaban
yang tepat adalah C.

Latihan Soal:
Dear Ima,
Let me introduce myself. My name is Adiba Salma Zakiya, just call me Salma. I’m
from Bima, Nusa Tenggara Barat. I read your profile in NONA magazine and would like to
be one of your friends. It’s a bit hard to find internet connection here, so I decided to write
a letter insread of an email to you. I hope you dont mind.
I’m the first child in my family. My parents have four children. I got to SMAN 3 Bima.
Now, I’m in the 10th Grade. My hobby is cooking, that’s why I can cook many kind of
Bimanese traditional dishes. Beside, I also like to play Kareku Kandei. It’s a traditional
music performance from Bima. Girls pound their pestles to a mortar to create harmonious
sound. Kareku Kandei is usually performed on special events such as Independence Day,
Kartini Day, etc.
I would love to know you better, would you tell me about yourself too?
Regards,

Salma

32
1. How did Salma know about Ima’s profile?
A. By writing a letter to Ima
B. By sending an email to Ima
C. By reading her profile in a magazine
D. By being one of her friends
E. By listening to her profile on the radio

2. What is the main idea of paragraph one?


A. Salma described her hometown
B. Both salma and ima liked NONA magazine
C. Salma found Ima’s profile in NONA magazine
D. Salma complained about the internet connection
E. Salma described her family

LATIHAN SOAL UN 2017


20 This text is for questions 20 to 22. Disajikan teks tulis
berbentuk surat
JI. Jambu 129 pribadi (personal
Bandung letter), peserta didik
dapat menentuikan
30th January 2017 pernyataan yang
tepat terkait
hubungan antara
Dear Fred, pengirim dan
penerima surat
It was a real sorrow that I heard this morning of your great loss. I knew
your mother was ill. For your brother told me several weeks ago.
However, as he at that time did not seem to think the illness was very
serious, the news of your mother’s death came to me as a shock. You
have my sincere and heartfelt sympathy, my dear fellow, in your
sorrow.

I know you will feel it deeply, for you always thought so much of your
mother and loved her so truly. I feel it also as a personal loss to myself
for your mother was always very kind to me. I admired her a good and
noble woman. Her death must be a terrible grief to your father too.
Please assure him of my sincere sympathy.

Words, I know, are poor comforters. “The heart knows its own sorrow”,
and in such sorrows we are always alone. However, it is not mere
words when I say that I feel with you in your sorrow.

Your sincere friend,

20. What is the possible relation between the sender and the recipient?

A. Relatives.

B. Employers.

C. Friends.

D. Siblings.

E. Employees.

21 21. From the letter we know that... Disajikan teks tulis


berbentuk surat
A. Fred’s mother was not ill before her death. pribadi (personal
B. Jack sent letter to Fred several weeks ago. letter), peserta didik
dapat menentuikan
C. Jack was very sorrowful to send the letter to his mother. pernyataan yang
tepat terkait dengan
D. Jack had known Fred’s mother before.
isi pesannya
E. Fred is the only child in his family.

33
22 22. “You have my sincere and heartfelt sympathy, my dear fellow, in Disajikan teks tulis
your sorrow.” What is the closest meaning of the underlined word? berbentuk surat
pribadi (personal
A. Real. letter), peserta didik
B. Caring. dapat menentukan
kata yang tepat
C. Generous. yang memiliki
kesetaraan makna
D. Honest.
dengan kata yang
E. Curious. digaris bawahi
pada kalimat dari
teks.

Menentukan gambaran umum atau informasi tertentu/rinci/tersirat atau rujukan kata atau
makna kata/frasa dari teks tertulis berbentuk advertisement/brochure.

2. ADVERTISEMENT/BROCHURE

Contoh soal dan pembahasan

Urgently needed !!

A developing foreign company looks for PRODUCTION MANAGER for its office products
division. Candidate must have at least 7 years experience in production development and
management in the field and the ability to motivate and train incoming production staff.
Good salary and benefits package. Qualified candidates should send their resumes to:
The Bright News, Box 8552, 1627 Elm Street, Adelaide, Australia.

1. The advertisement is about……

a. Job vacancy d. Properties

b. Imported Furniture e. Holiday package

c. Cloths and shoes sales

Pembahasan: pada iklan diatas, kalimat pertama menunjukkan company looks


(perusahaan mencari) production manager. Sehingga jawaban yang tepat adalah a. Job
vacancy (lowongan pekerjaan).

2. Interested applicant does not need to have……

a. at least 7 years experience d. skill to train production staff.

b. good management in the field e.S1 degree with at least GPA 3.00

c. ability to motivate others


Pembahasan: pada iklan diatas, dimulai dari kalimat kedua merupakan kualifikasi
yang harus dimiliki oleh pelamar, kecuali e. S1 degree with at least GPA 3.00.

34
LATIHAN SOAL. Read the text then answer the questions.

1. Thos who succesfully complete the course will be given...


A. Free tuition
B. A further training course
C. Jobs in big hotels in the colony
D. A chance to stay in the colony’s leading hotels for one month
E. Membership of the hongkong institute of Hotel Management

2. What requirement is needed by an applicant to apply for the hotel work training?
A. Able to speak Mandarin
B. Secondary school graduate
C. Good knowledge of English
D. Reutable uniersity graduate
E. 2-year expereince in hotel work

Menentukan gambaran umum atau informasi tertentu/rinci/tersiratt atau rujukan kata atau
makna kata/frasa dari teks tertulis berbentuk Announcement.

3. ANNOUNCEMENT
Pengumuman (announcement) merupakan informasi tentang sesuatu yang telah
terjadi atau akan terjadi yang disampaikan kepada public. Teks pengumuman terdiri
dari 4 bagian:
 Head (judul pengumuman)
 Body atau content (berisi informasi rinci tanggal, tempat dan program yang menjadi
pokok informasi)
 Closing (for your information….)
 Writer (penulis atau yang bertanggungjawab terhadap pengumuman tersebut.

35
Contoh soal dan pembahasan

Extracurricular Activity
Rohis Mentoring
 Available Monday – Friday (from 15.00 – 17.00 WIB)
 Spiritual activity for all students
 Experienced mentor
 Open for all students of SMAN 10 Depok grade X dan XI
 No fee
 Bring you mushaf
To enroll please contact Mr. Erwan or come to mosque every day after Dzuhur
praying between 12.00 – 13.00 WIB.

1. What is the announcement for?


a. Rohis d. The alumni of SMAN 10 Depok
b. Mentoring e. Students of grade X dan XI
c. All students of SMAN 10 Depok

Pembahasan: Didalam pengumuman, disebutkan bahwa yang dapat mendaftar kegiatan


mentoring rohis hanya siswa kelas X dan XI. Sehingga jawaban yang tepat adalah E.

2. Which of the following statement is true?


a. The rohis mentoring program is compulsory
b. The rohis mentoring program is held seven days a week
c. The activity will train students’ physical body
d. The mentoring program is pretty expensive
e. Only student of grade X and XII can register

Pembahasan: Pilihan merupakan pilihan yang salah karena kegiatan mentoring tidak
diwajibkan. Pilihan B juga salah karena mentoring dibuka hanya 5 hari. Pilihan C juga
salah karena kegiatan ini merupakan kegiatan spiritual. Pilihan D tidak tepat karena
kegiatan mentoring bebas biaya (no fee). Pilihan yang benar adalah E.

LATIHAN SOAL
This text questions 18 and 19. Disajikan teks tulis berbentuk undangan
resmi, peserta didik dapat menentukan
pernyataan yang tepat terkait dengan
We are announcing today that we are bringing the topiknya
Milestone and Ever Green brands even closer together.
Effective as of 5 December 2013, our official name will
be:
GREEN MILES WEST
The substitution of “West” in our name---replacing
“Cianjur”-—is the result of an agreement we reached with
Cianjur Gardening Association, following a protest over
the original use of ”Cianjur” in our name.
We hope this does not create any confusion among our
loyal consumers. While this represents a change from
our initial name introduction, it does not change the
quality of products we offer to our consumers.

18. Who issues the announcement?


A Green Miles West Company.

36
B. Green Miles West Customers.
C. The co-worker of Green Miles West.
D. The partnership of Green Miles West.
E. The loyal customers of Green Miles West.

19. “The substitution of “West” in our name...“. disajikan teks yang sama tentang undangan
resmi, peserta didik dapat menentukan kata yang
The word “substitution” has closest in meaning to .... tepat yang memiliki kesetraan makna dengan
A. merger kata yang bergaris bawah pada kalimat dari teks

B. insertion
C. development
D. replacement
E. improvement

37
B. TEXT TYPES

Menentukan gambaran umum atau informasi tertentu/rinci/tersirat atau rujukan kata atau
makna kata/frasa dari teks tertulis berbentuk News Item.

1. NEWS ITEM
News item is a text which informs readers about events of the day. The events are
considered newsworthy or important.
Generic Structure of News Item
1. Main event
2. Elaboration (background, participant, time, place)
3. Resource of information
Language Feature of News Item
1. Focusing on circumstances
2. Using material process

Contoh Soal dan Pembahasan News Item


A. Indonesian Maid in HK Court after Having Sex
About 500 ancient masks from many regions across Indonesia are being
displayed at the Sonobudoyo Museum in Yogyakarta from Nov. 20 to 29.
The event, called The Power of Topeng (The Power of the Mask), presents 400
masks that are simulacrums of statues from all over the country and the other masks
that are from the collection of the Sonobudoyo Museum.
The curator of the event, I Wayan Dana, said that based on Indonesian history,
masks not only functioned as works of art, but also as instruments of religious rituals.
“The two functions strengthen the mask's position as having strong roots in
Indonesian cultures,” he said when speaking at the event's opening ceremony on
Thursday night.
In religious rituals, he said, masks were used as instruments to communicate
with supernatural powers, while their shapes were adjusted to the traditions in each
era. He added that the masks also developed side by side with the changes of
kingdoms in this archipelago.
Van Heekeren, the author of Prehistoric Life of Indonesia, published in 1955,
noted that masks had been parts of Indonesian prehistory thousands of years ago. The
Mask of Panji, which developed during the Majapahit Kingdom, for example, told a love
story in the era.
Other examples are the Ramayana Mask, which was developed during the
Indonesian Hindu kingdoms, and the Ngayogyakarta Palace Mask that was made
during the era of Hamengkubuwono VII.
The head of Yogyakarta Culture Agency, Umat Priyono, expressed his
appreciation to the organizers of the event with the hope that it would widen people’s
understanding of the development of masks as a cultural product in Indonesia.

Contoh soal dan pembahasan:

1. As a part of religious rituals, what is the function of masks?


a. They are used as a means of communication.
b. They represent the changes of kingdoms in the era.
c. They are used among the kingdoms in the archipelago.
d. They function as adjusting the tradition and the religion.
e. They are used for communicating with supernatural power.

Pembahasan: Jawaban bisa dilihat pada paragraph ke 5 yaitu “In religious rituals, he said,
masks were used as instruments to communicate with supernatural powers”. Dari sini bisa
diambil kesimpulan bahwa jawaban yang tepat adalah a. They are used as a means of
communication

38
2. The second paragraph tells about....
a. the venue of the event
b. the purpose of the event
c. the statues from all over the country
d. the number of masks being presented
e. the collection of the Sonobudoyo Museum.

Pembahasan: pada paragraph ke 2 informasi yang didapat dalah jumlah topeng yang
dipamerkan. Sehingga jawaban yang tepat adalah d. the number of masks being
presented

3. A part from being artistic, in the past masks were considered …


a. scary d. sacred
b. funny e. expensive
c. exotic

Pembahasan: dari text tersebut bisa diambil kesimpulan bahwa topeng2 (masks) juga
digunakan sebagai instrument ritual keagamaan, sehingga masks (topeng2) juga
dianggap sebagai sesuatu yang d. sacred (keramat)

4. “ In religious rituals, he said, masks were used as instruments to communicate with


supernatural powers, while their shapes were adjusted to the traditions in each era.”
(Paragraph 5). The underlined word is closest in meaning to ….
a. Shown d. promoted
b. Created e. classified
c. Matched

Pembahasan: adjusted dapat diterjemahkan sebagai disesuaikan. Sehingga jawaban


yang mendekati adalah c. matched (dicocokkan).

LATIHAN SOAL NEWS ITEM

Thailand: Forty-two passengers were injured and two were missing after two boats
carrying Thai and foreign tourists collided in the popular island beach party, police said
Sunday.
The speedboats were ferrying tourists to the Pha-Ngan island for a “fullmoon”
party, a monthly event that attracts thousands of young, mostly western tourists.
Provincial police officer, Adipong Tapee said of the 42 travelers injured, 39 were
hospitalized including four Britons, four Australians and four Singaporeans. Also among
them were Irish, Norwegian, Malaysian and Thai tourists, he said.
The boats collided and overturned before midnight Saturday, throwing the
passengers into the rough sea water just off the island, the website of The Nation
newspaper said.
1. What’s the news about?
A. A full moon party on a tourist boat
B. A beach party in Pha-Ngan island
C. The missing tourist in Thailand
D. A collision between two boats
E. An accident on a speedboat

39
2. We know from the text that...
A. The boat carried forty two passengers
B. The accident happened not far from Pha-Ngan
C. The two missing passengers were westerners
D. The passengers were mostly foreign tourists
E. The boats collided on the way back to Pha-Ngan island

3. What had possibly caused the accident?


A. A careless captain
B. Too many passengers on board
C. There were too many boats
D. A rough weather
E. The boats were speeding

Menentukan gambaran umum atau informasi tertentu/rinci/tersirat atau rujukan kata atau
makna kata/frasa dari teks tertulis berbentuk Recount.

2. RECOUNT AND BIOGRAPHY


Recount is a text which retells events or experiences in the past. Its purpose is
either to inform or to entertain the audience. There is no complication among the
participants and that differentiates from narrative
Generic Structure of Recount
1. Orientation: Introducing the participants, place and time
2. Events: Describing series of event that happened in the past
3. Reorientation: It is optional. Stating personal comment of the
writer to the story
Language Feature of Recount
• Introducing personal participant; I, my group, etc
• Using chronological connection; then, first, etc
• Using linking verb; was, were, saw, heard, etc
• Using action verb; look, go, change, etc
• Using simple past tense

Example of Recount text


A. My Grandpa’s Funeral in Toraja
Last month my family and I went to Toraja to attend Grandpa’s funeral. It was my
first time to go to such a ceremony. We gathered there with our kin in the ceremony.
Overall, the ceremony was quite elaborate. It took about a week. Several days
before the ceremony was done, grandpa’s body was kept in a series of houses arranged
in a circular row around an open field called tongkonan. His corpse was dressed in a fi ne
wearing.
The funeral was performed in two phases. First, we slaughtered the pigs and
buffaloes, and then moved the corpse to face north. In this ceremony we wore black
clothes. After that, the corpse was placed in a sandal wood coffin. Then, it was brought
out of the house and placed on an open platform beneath the granary. Meanwhile, my
uncle, my brother, and I prepared the wooden puppet and a funeral tower called lakian.
The next phase of the ceremony was held in this place. The coffin is borne from the
house and placed in the lakian. During the day, there were also buffalo matches. They
were great matches. In the night, we were feasting, chanting, and dancing.
On the last day, the grandpa’s coffin were lowered from the funeral tower and
brought up to the mountain side family graveyard. It was followed by great shouting and

40
excitement from the relatives and the guests. Finally, we installed the wooden puppet on
a high balcony where other puppets representing the members of a whole family were
already there. The funeral ceremonies made my family and I tired. However, we were
grateful because it ran smoothly.

Soal dan Pembahasan:


1. When did the writer attend the funeral?
a. Last week d. Next Month
b. Last day e. Last month
c. Yesterday

Pembahasan: Jawaban bisa dilihat pada paragraph 1, sehingga jawaban yang tepat
adalah e. last month.

2. How long did the writer and his family hold the ceremony?
a. one week d. 3 days
b. several days e. 2 days
c. one month

Pembahasan: Jawaban bisa dilihat pada paragprah 2 yaitu a week. Sehingga jawaban
yang tepat adalah a. one month.

Latihan Soal Recount Text

Yesterday was the busiest day ever. My friend and I went to the campus to do
many academic activities there. In the morning, I spent time presenting a project given
last week, while my colleague assisted me, working behind the scene she operated the
slides and resumed the presentation.
Then we consulted the lecturer about our paper and our presentation. My
lecturer said that our presentation was quite satisfactory, but the paper was not. We had
to do a lot of corrections to the writing format, the choice of words, and the spelling. It
was shocking since we have composed it for over whole week. Surprisingly,we had to
hand the paper this evening.
Therefore, we revised the paper. We finished at 4 PM and soon we resubmitted
it. Then, we went to a meeting with Indonesia Mengajar’s club in the campus hall. We
discussed the teaching schedule for next month’s teaching programme.
It was really a busy day. I came home at 10 PM and went to sleep directly.

1. The writer have got a busiest day yesterday. He and his friend had to revise their
paper that they had written for over a week. What would have happened to them if
they didn’t revise the paper yesterday ?
a. The paper wouldn’t finish so the writer would fail his study
b. His lecturer would serve notice to his lately submitted paper
c. The writer would feel frustation because he failed to finish his paper
d. The writer would ask his lecturer to rescheduled submitting the paper
e. The paper wouldn’t have finished at that time so the writer would have failed his
study

41
LATIHAN SOAL RECOUNT/BIOGRAPHY UN 2017
16 This text is for questions 16 and 17. Disajikan teks
berbentuk recount
Luis Lionel Andres Messi, born June 24, 1987, is an
tentang
Argentina football player for F.C Barcelona. He is not very
pengalaman./peristi
tall, mainly, due to a growing problem he had when he was
wa/ kejadian,
younger. His eyes are brown. He never has short hair.
peserta didik dapat
Lionel Messi started playing football at a very early age in menentukan
his hometown’s Newell’s Old Boys. From the age of 11, he informasi rinci di
suffered from a hormone deficiency and as Lionel’s dalam teks
parents were unable to pay for the treatment in Argentina,
they decided to15 move to Barcelona, Spain.
In the 2003-2004 season, when he was still only 16 years
old, Messi made his first team debut in a friendly match
with Porto that marked the opening of the new Dragao
stadium.
The following championship-winning season, Messi made
his first appearance in an official match on October 16,
2004, in Barcelona’s derby win against at the Olympic
Stadium (0-1). And now, in 2010, 2011, and 2012 he are
best player in the world with collect 3 Ballon D’Or

16. According to the text. Messi’s parents moved to


Barcelona ....
A. because they were very poor in Argentina
B. because they wanted Messi to be successful in
soccer
C. so that Messi could learn in the best soccer club
D. to get Messi’s health problem cured
E. to find the best treatment to cure Messi’s health
problem

17 17. What is the main idea of the third paragraph? Disajikan teks
berbentuk recount,
A. He hasn’t really attractive face, but he’s a very good
peserta didik dapat
football player.
menentukan
B. Lionel Messi is a good player for FC. Barcelona. pernyataan yang
tepat terkait satu
C. His best characteristics are on the foot has unsur
competitiveness. pengalaman/peristiw
D. He appears to be a quite good and modest person. a/kejadian di
dalamnya.
E. He has long black hair and brown eyes.

35 This text is for questions 35 to 38. Disajikan teks tulis


biografi pendek dan
Galileo Galilei (1564-1642) — Astronomer and Scientist.
sederhana tentang
Galileo developed a superior telescope and made many
tokoh ilmuwan,
significant discoveries in astronomy. He was sentenced to
peserta didik dapat
life imprisonment by the inquisition for his support for the
menentukan
Copernican theory that the sun was at the centre of the
pernyataan yang
solar system.
tepat terkait hal yang

42
Galileo was born in Florence, Italy in 1564 to a poor but dapat dipelajari dari
noble family. His parents recognised their child’s innate biografi tersebut
intelligence and talents and so made sacrifices to have
him educated. At his father’s insistence, Galileo studied
the profitable career of medicine. But, at the University of
Pisa, Galileo became fascinated in a wide range of
subjects. He was also critical of many of Aristotle’s
teaching which had dominated education for the past
2,000 years.
Galileo was appointed to be a mathematics professor at
the University of Pisa, but his strident criticisms of Aristotle
left him isolated among his contemporaries. After three
years of persecution, he resigned and went to the
University of Padua, where he taught maths. His
entertaining lectures attracted a large following and he was
able to spend the next 18 years pursuing his interests in
astronomy and mechanics.
35. Something that we can learn from Galileo Galilei’s
biography is ....
A. we must always believe whatever the society believes
to avoid getting persecuted
B. we must never go against the believe of the society to
avoid getting imprisoned
C. we should believe in something and stay faithful to it no
matter how hard
D. we must sacrifice everything to get ourselves educated
E. we should spend 18 years to pursue our interests

36 36. From the fact that Galileo Galilei was imprisoned for Disajikan teks tulis
supporting the Copernican theory, which later was proven to be biografi pendek dan
true, we know that Galileo Galilei was . ... sederhana tentang
tokoh ilmuwan yang
A. reckless
sama, peserta didik
B. stubborn dapat menentukan
pernyataan yang tepat
C. tenacious terkait tindakan
D. arrogant keteladanan tokoh
biografi tersebut
E. foolish

37 37. According to the biography, why did Galileo Galilei resign Disajikan teks tulis
from his teaching job at the University of Pisa? biografi pendek dan
sederhana tentang
A. He had attracted many followers.
tokoh ilmuwan yang
B. He was persecuted for three years. sama, peserta didik
dapat menentukan
C. He was appointed as a mathematics professor. pernyataan yang tepat
D. He had a better offer from the University of Padua. terkait pengalaman
hidup tokoh
E. He wanted to pursue his interests in astronomy and
mechanics.

38 38. “His parents recognised their child’s innate intelligence and Disajikan teks tulis
talents and so made sacrifices to have him educated.” (Paragraph biografi pendek dan

43
2) sederhana tentang
tokoh ilmuwan yang
The underlined word is closest in meaning to . ...
sama, peserta didik
A. acquired dapat menentukan kata
yang tepat yang
B. ingenious memiliki kesetaraan
C. multiple makn dengan kata yg
digarisbawahi pada
D. inborn kalimat dari teks
E. strident

Menentukan gambaran umum atau informasi tertentu/rinci/tersirat atau rujukan kata atau
makna kata/frasa dari teks tertulis berbentuk Explanation.

3. EXPLANATION
Explanation is a text which tells processes relating to forming of natural, social,
scientific and cultural phenomena. Explanation text is to say 'why' and 'how' of the
forming of the phenomena. It is often found in science, geography and history text
books.
Generic structure of Explanation
 General statement; stating the phenomenon issues which are to be explained.
 Sequenced explanation; stating a series of steps which explain the
phenomena.
Language Feature
 Featuring generic participant; sun, rain, etc
 Using chronological connection; to begin with, next, etc
 Using passive voice pattern
 Using simple present tense

Example of Explanation Text

Icebergs are mountains of freshwater ice floating in the ocean. They are huge chunks
broken off from the great masses of land ice called glaciers.

Almost all of Greenland and Antarctica are covered by glaciers the year round. Glaciers
also cover parts of Alaska. They are formed by layers of packed snow. Glacier may be
thousands of meters thick. Their front ends, or tongues, reach down to the sea. At the
coast the tips of the tongues break off, plunge into the oceans, and become icebergs. This
process is called calving.

When calving occurs, a loud cracking noise fills the air. Sometimes a low rumbling can be
heard for hours before the ice actually breaks away. People close enough can hear the
hissing of air as it escapes from bubbles bursting in the ice along the break.

Glaciers calve all year round. Just as many icebergs break off in winter as in summer. But
in winter their passageway to open the sea is often jammed with masses of frozen
seawater. Icebergs pile up behind this jam of sea ice. In the spring, when the ice block is
broken, a whole fleet of icebergs may sail out toward the open ocean.

Icebergs from the eastern coast of Greenland drift southward. They are carried by the
Greenland Current, which then swings them northward around the tip of the island. Part of
the way up the coast the icebergs are caught in the cold Labrador Current and carried
southward toward Newfound-land

On the way, most of the icebergs become grounded among the many islands and bays
along the Labrador coast. The others float on toward the open sea. Off Newfoundland they

44
are caught by a warm current from the south called the Gulf Stream. Icebergs that do not
ground and remain in the Labrador Current often enter the lanes used by ships crossing
the Atlantic Ocean.

An iceberg starts to break up almost as soon as it is afloat. Cracks appear and become
filled with water from ice that melts during the day. When this melted water freezes at
night, it expands and widens the cracks. The ice is weakened, and pieces of iceberg break
off and float away. The iceberg becomes smaller and smaller. Most icebergs melt
completely within a few days of entering the Gulf Stream

Contoh soal dan pembahasan:


1. What does the text tell us about?
a. What calving is d. How icebergs exist
b. What glaciers cover e. How iceberg is defined
c. How far glaciers cover

Pembahasan: Jawaban dari pertanyaan in bisa dilihat dari paragraph utama dari artikel ini
di paragraph 1. Kemudian paragraph selanjutnya adalah pembahasan tentang proses
terbentuknya iceberg. Sehingga bisa diambil kesimpulan jawaban yang tepat adalah d.
How icebers exist (bagaimana terbentuknya iceberg)

2. The purpose of this text is….


a. to describe Iceberg in detail
b. to give readers the information of Iceberg
c. to tell about the area covered with huge Icebergs
d. to explain the process of iceberg as a natural phenomenon
e. to retell natural events for the purpose of informing or entertaining

Pembahasan: Tujuan komunikatif explanation text adalah to explain (untuk menerangkan)


process of how things work or happen. Sehingga jawaban yang tepat adalah d.

Latihan soal Explanation


There are millions of plants and animals living in the sea. Most of the plants and animals
living in the oceans are extremely small and float near the surface of the water. They are
food for huge numbers of small animals that also live near the surface. Together, they are
all known as plankton.

Many fish feed on this plankton, including one of the largest, the whale shark. Despite their
great size, certain whales live entirely on plankton. The blue whale is the largest animal
ever to live. It grows to a length of over 100 feet. The humpback whale is a smaller kind
and has long flippers. It can jump right out the water. The Californian grey whale makes
long migrations every year – it spends the summer feeding in the Artic but swims south to
the warmer waters off the coast of Mexico in the winter. The females give birth in the
shallow, warm water there.

Many of the fish in the sea may be eaten by other meat-eaters. Sharks, for example, are
some of the fastest hunters, although not all of them eat other fish.

Deep down in the oceans live many strange fish. It is so dark down there that many of
these fish have “light” on them, which are used for attracting smaller fish for the larger
ones to eat.

45
Jellyfish are peculiar-looking creatures ranging in size from a fraction of an inch to 6 feet
across. They don’t really swim but drift in the currents of the open oceans. Although they
contain a powerful sting, they are often eaten by turtles. Turtles are reptiles that spend
most of their eggs on sandy beaches.

Question:
Choose one of the following statements which is correct based on the text above.
A. Jellyfish being sea creatures are food for other animals.
B. All sharks being felonious hunters in ocean live on plankton.
C. Millions of creatures living on land as turtles and sharks eat plankton.
D. The fish having light on them will make use of the deep ocean to hunt their prey at
night.
E. The plants and animals floating on the surface of ocean provide food for other
creatures to save themselves.

LATIHAN SOAL UN 2017


39 This text is for questions 39 to 42. Disajikan teks
tulis ilmiah
A tsunami is a series of ocean waves that sends surges of water,
faktual (factual
sometimes reaching heights of over loo feet (30.5 meters), onto land.
report) tentang
These walls of water can cause widespread destruction when they
orang /binatang/
crash ashore.
benda /binatang
These awe-inspiring waves are typically caused by large, undersea / gejala dan
earthquakes at tectonic plate boundaries. When the ocean floor at a peristiwa alam
plate boundary rises or falls suddenly, it displaces the water above it dan sosial
and launches the rolling waves that will become a tsunami. terkait mata
pelajaran lain di
Most tsunami, about 80 percent, happen within the Pacific Ocean’s kelas XII,
“Ring of Fire,” a geologically active area where tectonic shifts make peserta didik
volcanoes and earthquakes common. dapat
Tsunamis may also be caused by underwater landslides or volcanic menentukan
eruptions. They may even be launched, as they frequently were in pernyataan
Earth’s ancient past, by the impact of a large meteorite plunging into yang tepat.
an ocean.
Tsunamis race across the sea at up to 500 miles (805 kilometres) an
hour — about as fast as a jet airplane. At that pace they can cross
the entire expanse of the Pacific Ocean in less than a day. And their
long wavelengths mean they lose very little energy along the way.
A tsunami’s trough, the low point beneath the wave’s crest, often
reaches shore first. When it does, it produces a vacuum effect that
sucks coastal water seaward and exposes harbour and sea floors.
This retreating of sea water is an important warning sign of a
tsunami, because the wave’s crest and its enormous volume of water
typically hit shore five minutes or so later. Recognizing this
phenomenon can save lives.
A tsunami is usually composed of a series of waves, called a wave
train, so its destructive force may be compounded as successive
waves reach shore. People experiencing a tsunami should
remember that the danger may not have passed with the first wave
and should await official word that it is safe to return to vulnerable
locations.

46
Some tsunamis do not appear on shore as massive breaking waves
but instead resemble a quickly surging tide that inundates coastal
areas.
39. Why did the author write the report?
A. To raise people’s awareness about tsunami.
B. To warn people about an upcoming tsunami.
C. To inform people about past tsunamis.
D. To inform people about the different types of tsunami.
E. To warn people about the dangers of tsunami.

40 40. Tsunamis are usually the result of ... Disajikan teks


tulis ilmiah
A. the sudden rise or fall of ocean floors
faktual (factual
B. Pacific Oceans “Ring of Fire” report) tentang
orang /binatang/
C. the awe-inspiring waves benda /binatang
D. volcanoes / gejala dan
peristiwa alam
E. landslides dan sosial,
peserta didik
dapat
menentukan
pernyataan
yang tepat
terkait dengan
jenis-jenis
informasi yang
tercakup dalam
perincian
tentang
obyeknya.

41 41. From the text, we know that Tsunami can be very destructive Disajikan teks
because.... tulis ilmiah
faktual (factual
A. they come after earthquakes
report) tentang
B. they are caused by volcanic eruptions orang /binatang/
benda /binatang
C. they are tall, fast, forceful and repetitive / gejala dan
D. they have a vacuum effect peristiwa alam
dan sosia,
E. they occur suddenly peserta didik
dapat
menentukan
pernyataan
yang tepat
terkait dengan
deskripsi
obyeknya

42 42. “Some tsunami do not appear on shore as massive breaking Disajikan teks
waves but instead resemble quickly surging tide that inundates tulis ilmiah
coastal areas.” (Paragraph 8) faktual (factual
report) tentang
The underlined word is closest in meaning to ....
orang /binatang/
benda /binatang

47
A. floods / gejala dan
peristiwa alam
B. covers
dan sosial,
C. fills peserta didik
dapat
D. attacks menentukan
E. submerges kata yang tepat
ysng memiliki
kesetaraan
makna dengan
kata yang
digarisbawahi
pada kalimat
dari teks

Menentukan gambaran umum atau informasi tertentu/rinci/tersirat atau rujukan kata atau
makna kata/frasa dari teks tertulis berbentuk Exposition.

4. ANALYTICAL EXPOSITION
Exposition is a text that elaborates the writer‘s idea about the phenomenon
surrounding. Its social function is to persuade the reader that the idea is important
matter.
Generic Structure of Analytical Exposition
 Thesis: Introducing the topic and indicating the writer’s position
 Argument 1: Explaining the argument to support the writer’s position
 Argument 2: Explaining the other arguments support the writer’s position
more
 Reiteration: Restating the writer’s position
Language Features of Analytical Exposition
 Using relational process
 Using internal conjunction
 Using causal conjunction
 Using Simple Present Tense

Example of Analytical Exposition


A. Is Smoking Good for Us?
Before we are going to smoke, it is better to look at the fact. About 50 thousand
people die every year in Britain as direct result of smoking. This is seven times as
many as die in road accidents. Nearly a quarter of smokers die because of diseases
caused by smoking.
Ninety percent of lung cancers are caused by smoking. If we smoke five
cigarettes a day, we are six times more likely to die of lung cancer than a non smoker.
If we smoke twenty cigarettes a day, the risk is nineteen greater. Ninety five percent
of people who suffer of bronchitis are people who are smoking. Smokers are two and
half times more likely to die of heart disease than nonsmokers.
Additionally, children of smoker are more likely to develop bronchitis and
pneumonia. In one hour in smoky room, nonsmoker breathes as much as substance
causing cancer as if he had smoked fifteen cigarettes.
Smoking is really good for tobacco companies because they do make much
money from smoking habit. Smoking however is not good for everybody else.

Contoh Pertanyaan dan pembahasan.

1. What’s the main idea of the text above?


a. Ninety percent of lung cancers are caused by smoking

48
b. Children of smoker are more likely to develop bronchitis and pneumonia
c. Smoking is really good for tobacco companies because they do make much
money from smoking habit
d. Some facts related to effects of smoking
e. About 50 thousands people die every year in Britain as direct result of
smoking

Pembahasan: Kita dapat mendapatkan main idea dari text exposition diatas
dengan melihat ide – ide pokok pada setiap paragraph. Ide pokok pada setiap
paragraph menceritakan tentang akibat dari merokok (effect of smoking), sehingga
jawaban yang tepat adalah d. some facts related to effects of smoking.

2. According to the text above, why children of smokers are most likely to have
diseases from smoking?
a. Because ninety five percent of people who suffer of bronchitis are people
who are smoking
b. Because nearly a quarter of smokers die because of diseases caused by
smoking
c. Because smokers are two and half times more likely to die of heart disease
than non-smokers
d. Because in one hour in smoky room, nonsmoker breathes as much as
substance causing cancer as if he had smoked fifteen cigarettes.
e. Because if we smoke twenty cigarettes a day, the risk is nineteen greater
Pembahasan:

Pada paragraph 3 membicarakan tentang pengaruh orang tua yang merokok pada anak
– anak. Di kalimat kedua merupakan akibat dari smoky room (ruang yang berasap), oleh
karena itu jawaban yang tepat adalah d. Because in one hour in smoky room, non
smoker breathes as much as substance causing cancer as if he had smoked fifteen
cigarettes.

B. Laptop as Students' Friend


Conventionally, students need book, pen, eraser, drawing book, ruler and such
other stuff. Additionally, in this multimedia era, students need more to reach their
progressive development. Students need mobile keyboards to record every presented
subject easily. Of course it will need more cost but it will deserve for its function.
First, modern schools tend to apply fast transferring knowledge because the school
needs to catch the target of curriculum. Every subject will tend to be given in
demonstrative method. Consequently students need extra media cover the subject. Since
there is a laptop on every student’s desk, this method will help student to get better
understanding.
Secondly, finding an appropriate laptop is not difficult as it was. Recently there is an
online shop which provides comprehensive information. The best is that the shop has
service of online shopping. The students just need to browse that online shop, decide
which computer or laptop they need, and then completes the transaction. After that the
laptop will be delivered to the students' houses. That is really easy and save time and
money.
From all of that, having mobile computer is absolutely useful for students who want
to catch the best result for their study. Buying laptop online is advisable because it will cut
the price. This online way is recommended since online shop also provides several laptop
types. Students just need to decide which type they really need.

1. What is the purpose of the text?


a. To persuade the reader about laptop as student’s friend
b. To describe about laptop as student’s friend
c. To entertain about laptop as student’s friend
d. To report about laptop as student’s friend

49
e. To retell about laptop as student’s friend

Pembahasan: pada soal diatas yang ditanyakan adalah purpose (tujuan) dari text.
Fungsi (function) dari analytical exposition adalah untuk membujuk atau persude. Jadi
jawaban yang tepat adalah A.

2. Why modern school tend to apply fast transferring knowledge?


a. Because it follow the modern technology
b. Because it is a self-regulation
c. Because it is the provision of education office
d. Because it is obligatory on the modern school
e. Because it needs to catch the target curriculum

Pembahasan: jawaban dari pertanyaan diatas dapat dilihat pada paragraph ke 3 yaitu
E. Because it needs to catch the target curriculum.

3. Who need extra media cover in the subject?


a. Teachers d. University Student
b. Children e. Professor
c. Students

Pembahasan: jawaban dari pertanyaan diatas dapat dilihat pada paragraph ke 2 baris ke
3 yaitu students need extra media cover the subject. Sehingga jawaban yang tepat
adalah C. Students.

LATIHAN SOAL ANALYTICAL EXPOSITION

Do you agree that music is important in our life? Yes I do, music has certain role
completing our day to day activities. Here are some reasons why music is heard
everywhere and anywhere.

Music is a way to express feelings. When we fall in love, the kind of music we’d listen to
would be all about love. When we’re sad, we would go for music that is melancholic in
nature and immerse ourselves in the sadness. When we’re happy, we’d choose songs with
happy tunes too.

Song can help to memorize the last experiences. A favorite song is a powerful
documentary. People with Alzheimer which are impaired the brain would remember details
about songs they were familiar with. For example, an elderly woman who couldn’t even
remember her husband’s name would remember the details of her favorite song; when it
was played, how it made her feel and things about the song that made it especially
memorable for her.

Further, music can unite people for a cause and changes the world. A song with good lyric
and striking deep chord can stimulate the universal feeling of all people. We can see it in
the case of the famous and legendary Michael Jacson’s Heal the World. It can arouse
humanism of a lot people in this world.

So what would the world be like without music? It would be lonely.


Adapted from The Jakarta Post

50
Q: What’s the topic of the text?
A. How to compose a piece of music
B. Diffrent kinds of music
C. The importance of music
D. Music is one of the most important aspects in a play
E. How to become a musician

LATIHAN SOAL UN 2017

43 Pakistan’s houbara bird facing extinction Disajikan teks tulis


berbentuk eksposisi analisis
The indiscriminate hunting of endangered houbara bustard tentang topik yg hangat
birds in Pakistan by rich hunting parties from the Gulf dibicarakan , peserta didik
nations have pushed the species toward extinction. The dapat menentukan
hunting was initially banned by the Pakistani courts, but pernyataan yang tepat
under continuous pressure from the provincial governments terkait dengan topiknya
of Sind, Baluchistan, and Punjab, as well as the federal
government, recently the courts lifted the ban on the hunting
of this vulnerable species.

Some people in Pakistan have been advocating for


sustainable hunting of the species instead of a complete
ban, but this is highly debatable. Reckless hunting for
decades has pushed this beautiful species toward extinction
and it is now critically endangered in Pakistan.

No reliable or credible statistics on the population dynamics


of the species are available either. Unless specially
protected by law, the species runs the risk of becoming
extinct in the wild in the next few decades.

Under these circumstances, the only logical approach to


save the species from extinction in the not-so-distant future
is to immediately start comprehensive captive breeding
program for the species across the nation.

If the captive breeding program runs successfully and


produces an appreciable number of chicks, then
reintroduction programs could be initiated in their wild
habitats.

Only after rebuilding the population for some time in the wild
and allowing them to breed naturally to reach a sustainable
population could some restricted hunting be planned.
Currently, the low numbers have made this species
vulnerable and with zero or poor monitoring, sustainable

51
hunting practices in Pakistan for this dwindling species will
not work.

Game hunting works only when a stable population is


achieved with strict monitoring on the ratio of males to
females hunted per season. If the females (no true statistics
available) are being taken out of the population through
random hunting, the population is sure to pass through a
bottleneck in the wild habitat, with high vulnerability to
eventual extinction.
The people of Pakistan need to decide what is more
important-a few petrodollars or protecting the biodiversity of
the nation.
Saikat Kumar Basu
Lethbridge, Canada

43. What is the subject of the text?


A. The extinction of Pakistan’s houbara bird.
B. The hunting of Pakistan’s houbara bird.
C. The conservation of Pakistan’s houbara bird.
D. The rehabilitation of Pakistan’s biodiversity.
E. The protection of Pakistan’s biodiversity.

44 44. From the text, we know that the writer . ... Disajikan teks tulis
A. enjoys hunting the houbara bird berbentuk eksposisi analisis
B. advocates for sustainable houbara bird hunting tentang topik yg hangat
C. approves all kinds of houbara bird hunting dibicarakan , peserta didik
D. tries to persuade people to conserve houbara bird dapat menentukan
E. breeds houbara bird in captivation pernyataan yangb tepat
terkait pandangan atau
pendapat penulis
45 45. According to the text, why does the writer want all kinds Disajikan teks tulis
of houbara bird hunting be banned? berbentuk eksposisi analisis
A. The population of houbara bird is getting smaller and tentang topik yg hangat
smaller. dibicarakan , peserta didik
B. The captivation of houbara bird has been unsuccessful. dapat menentukan
C. The houbara bird is difficult to catch. pernyataan yangb tepat
D. The houbara bird is beautiful. terkait dengan argumentasi
E. The hunters are reckless. pendukungnya

46 46. From the passage, we can conclude that people can Disajikan teks tulis
resume hunting the houbara bird only ... a sustainable berbentuk eksposisi analisis
population is reached. tentang topik yg hangat
A. after dibicarakan , peserta didik

52
B. when dapat menentukan kata
C. because sambung yang tepat untuk
D. if melengkapi pernyataan
E. before terkait dengan isi pesannya

5. HORTATORY EXPOSITION

Seperti halnya Analytical Exposition, Hortatory Exposition adalah jenis teks


bahasa Inggris yang tergolong ke dalam kelas Argumentation.Hortatory
Exposition adalah sebuah jenis teks bahasa Inggris yang mana menghadirkan usaha
kami memengaruhi pembaca untuk melakukan sesuatu atau bertindak dalam hal
tertentu. Dalam Hortatory Exposition, kami mencantumkan beberapa pendapat
mengenai hal tertentu untuk memperkuat ide pokok dari teks tersebut.
Berbeda dengan Analytical Exposition, Hortatory
Exposition menghadirkan Recommendation sebagai paragraf penutup dari
sebuah Hortatory Exposition Text.
Dalam, Recommendation ini, berusaha mengajak dan membujuk pembaca untuk
melakukan sesuatu. Hal ini jauh berbeda dengan Analytical Exposition yang mana
menempatkan Reiteration atau kajian kembali ide pokok sebuah teks sebagai penutup
tanpa adanya ajakan atau bujuakan kepada pembaca.

Tujuan Komunikatif Hortatory Exposition Text

Berbeda dengan Analytical Exposition Text yang tujuan komunikatifnya


memaparkan dan memengaruhi pembaca ( to explain and influence the reader )
bahwa kejadian yang di ceritakan itu penting, pada Hortatory Expostion Text tujuan
kommunikatifnya adalah memaparkan dan memengaruhi pembaca bahwa
seharusnya demikian dan seharusnya tidak demikian.

Struktur Kebahasaan Hortatory Exposition Text

Ada tiga bagian dari struktur atau pola kalimat dalam Hortatory Exposition Text, yaitu:

1. Thesis

Thesis berisi tentang pengenalan ide pokok kami tentang suatu gejala atau
kejadian yang akan diangkat atau dibahas.

2. Arguments
Arguments berisi tentang pendapat-pendapat yang mendukung ide pokok kami.
Semakin banyak pendapat yang kami tuliskan, semakin menarik sebuah Hortatory
Exposition Text itu, karena pembaca cenderung percaya terhadap suatu peristiwa jika
terdapat banyak pendapat yang mendukung di dalamnya.

3. Recommendation
Recommendation berisi tentang rekomendasi atau ajakan kami terhadap pembaca.
Ciri Kebahasaan Hortatory Exposition Text
Di bawah ini adalah ciri kebahasaan yang terdapat pada Hortatory Exposition Text,
yaitu:
1. Menggunakan simple present tense
2. Menggunakan temporal connectives; firstly, secondly, thirdly, etc.
3. Menggunakan evaluative words; importantly, valuably, trustworthy, etc.

53
Contoh Hortatory Exposition Text

OPEN LETTER TO THE PRIME MINISTER


Into the Mouth of Babes
We are writing to you because we are concerned about the way
Thesis food is being advertised to children.
What we eat now affects our health in years to come. Bad dietary
habits start while we are very young. For this reason, your government
supports health education that encourages a balanced healthy diet.
Australians are encouraged to get the bulk of their nutrients from fresh
Argument
fruits, vegetables, and wholegrain cereals, a lesser amount from foods
1
high in fat, salt and sugar.
TV advertising to children presents a completely different message.
Argument Nearly 80% of food advertising pushes fatty snacks or sweets – the very
2
foods that should be eaten least. To make matter worse, these ads take
up much of the advertising time. This must be stopped.
TV advertising is powerful and influential. Parents, however strong-
Argument willed, find it hard to resist pressures created by this advertising.
3 Children are least able to understand the tricks of the advertising
industry.

Recom- For the hours when children are the main audience, TV advertising
mendation of foods must be made to reinforce, not undermine, the message about
a balanced diet. We appeal to you, as Prime Minister, to take the lead in
calling together the advertisers, TV networks, consumers and public
health bodies to decide how this is to be done.

Yours sincerely,
Signed by Barbara Biggins and other prominent signatories.

Contoh soal dan pembahasan


1. What does the text tell us about?
a. To persuade the reader about advertisement for children
b. To persuade Prime Minister about advertisement for children
c. To persuade Prime Minister to handle a better advertisement for children
d. To report about advertisement for children
e. To retell about advertisement for children

Pembahasan: Tujuan komunikatif hortatory adalah memamaprkan dan mempengaruhi


pembaca tentang bagaimana seharusnya melakukan sesuatu. Sehingga jawaban yang
tepat adalah C.

2. What is the topic sentence of argument 2?


a. TV advertising to children presents a completely different message.
b. Nearly 80% of food advertising pushes fatty snacks or sweets
c. The very foods that should be eaten least.
d. To make matter worse, these ads take up much of the advertising time.
e. This must be stopped

Pembahasan: Jawaban dari pertanyaan ini bisa dilihat dari kalimat pertama pada
paragraph 3. Jawaban yang tepat adalah A.

54
Menentukan gambaran umum atau informasi tertentu/rinci/tersirat atau rujukan kata atau
makna kata/frasa dari teks tertulis berbentuk Review.

6. REVIEW TEXT

Pernahkah kalian mereview benda, film atau sebagainya? kalau belum pernah,
pernahkah anda melihat sebuah resensi film atau buku? Anda bisa lihat contoh Review
Text pada koran-koran yang menampilkan resensi film atau buku, sebagai gambaran apa
sih Review Text itu.
Mungkin sebagian orang pernah melihatnya bahkan pernah membuatnya. Review
Text adalah salah satu dari jenis teks bahasa Inggris (genre) yang ditujukan untuk
meninjau suatu karya baik berupa film, buku, benda dan lain sebagainya untuk
mengetahui kualitas, kelebihan dan kekurangan yang dimiliki karya tersebut yang
ditujukan untuk pembaca atau pendengar khalayak ramai.

Tujuan Komunikatif Review Text


Tujuan komunikatif dari Review Text adalah melakukan kritik terhadap peristiwa atau
karya seni untuk pembaca atau pendengar khalayak ramai, (critiques of the event or work
of art for the reader or listener masses), misalnya film, pertunjukan, buku, dll.

Generic Structure Review Text


Setiap jenis teks bahasa Inggris (genre) memiliki ciri Generic Structure yang berbeda-beda
sesuai dengan tujuan komunikatif yang akan disampaikan kepada pembaca. Dan
untuk Generic Structure dari Review Text, terdapat 4 struktur kalimat, yaitu:

1. Introduction
Seperti jenis teks bahasa Inggris (genre) yang lainnya, pada paragraph pertama dari
sebuah Review Text berisi tentang gambaran umum tentang sebuah karya atau benda
yang akan ditinjau. Gambaran umum tentang karya atau benda tersebut bisa berupa
nama, kegunaan, dan sebagainya.

2. Evaluation
Pada Generic Structure yang kedua, Evaluation, memberikan gambaran tentang detail
suatu karya atau benda yang direview, bisa berupa bagian-bagian dari karya atau benda
tersebut, keunikan dan kualitasnya. Tapi jangan terlalu memberikan banyak deskripsi
tentang karya atau benda tersebut, karena teks review kita seakan-akan "mengajarkan"
calon pembeli dan itu tidak bagus. Evaluasi karya atu benda sejauh yang dibutuhkan oleh
pembeli adalah yang paling benar. Istilah yang digunakan dalam sturktur paragraph yang
kedua ini tidak jauh dari kata "baik" atau "tidak" karya atau benda tersebut. Pada bagian
evaluasi, biasanya terdiri lebih dari satu evaluasi.

3. Interpretation
Pada bagian ketiga ini, kami memberikan pandangannya sendiri mengenai karaya atau
benda yang direviewnya. Tentunya phase ini dilakukan setelah melakukan evaluasi yang
cukup terhadap karya atau barang tersebut. Seringkali untuk mendukung dan memperkuat
pandangan kami mengenai karya atau benda yang direviewnya, kami tersebut
membandingkan karya atau benda tersebut dengan karya atau benda yang mirip. Pada
phase ini juga kami menuliskan di bagian apa karya atau benda tersebut bernilai atau di
bagian apa karya atu benda tersebut kurang bernilai.

4. Summmary
Pada bagian ini, kami memberikan kesimpulan kepada pembaca terhadap karya atau
benda yang telah direviewnya. Setelah memberikan penjelasan di evaluasi dan
pandangan penlis sendiri di interpretation, tibalah kami memberikan komentar apakah
karya atau benda yang direviewnya berharga atau tidak untuk calon pembeli.

55
Ciri Kebahasaan (Language Features)
Pada Review Text terdapat beberapa ciri kebahasaan seperti berikut ini:
1. Menggunakan present tense.
2. Banyak menggunakan adjective (kata sifat) seperti, bad, good, valuable, etc.
3. Sering menggunakan long clause (klausa panjang) dan kompleks.

Contoh Review Text

Harry Potter: Order of the Phoenix


Pengenalan / I absolutely love the Harry Potter series, and all of the books will
Orientasi always hold a special place in my heart.
Evaluasi 1 I have to say that of all of the books, however, this was not my
favorite.
Evaluasi 2 When the series began it was as much of a "feel good" experience as
a huge mug of hot cocoa. The stories were bright, fast-paced,
intriguing, and ultimately satisfying.
Tafsiran Order of the Phoenix is a different kind of book. In some instances
(Interpretative this works...you feel a whole new level of intensity and excitement by
recount) the time you get to the end. I was truly moved by the last page. Other
times the book just has a slightly dreary, depressing feel. The
galloping pace of the other books has slowed to a trot here, and parts
of it do seem long, as if we're reading all about Harry "just hanging
out" instead of having his usual adventures. Reading in detail about
Harry cleaning up an old house, for example - housekeeping is still
housekeeping, magical or no, and I'm not very interested in doing it
or reading about other people doing it.
A few other changes in this book - the "real" world comes much
more in to play rather than the fantasy universe of the previous
books, and Harry has apparently been taken off his meds. I know that
he had a lot to be grumpy in this book, especially with being a
teenager and all, but the sudden change in his character seemed too
drastic. He goes from being a warm-hearted, considerate person to
someone who will bite his best friend's heads off over nothing. It just
seemed like it didn't fit with his character, like he turned into a walking
cliché of the "angry teen" overnight.
Rangkuman The "real" story seemed to happen in the last 1/3 of the book, and
this part I loved. I actually liked the ending (and yes, I cried!) as sad
as it was. It packed a punch and it made me care about the story
even more. Still a really good book, with some editing it would have
been great.

Contoh soal dan pembahasan:

Reading the three plots of Marry Higgins Clark in All Through the Night concern a
stolen gold ornamental cup, a missing baby, and a will which seems to be deceitful. To
solve the mystery, she revives her beloved characters; Alvirah, the former cleaning
woman; and Willy Meeham, the plumber who won the lottery. They left their life in Jackson
Heights, Queens, for an apartment on Central Park. The two have fun along the way
solving the puzzle.
The pace is swift and the story is pure escape - totally fun Mary Higgins Clark. I
admit, however, that l am still trying to know what melody of the song "All Through the
Night" sounds like.

56
This holiday season, put aside your chores and curl up on couch with All Through
the Night. When you close it you will be relaxed and more ready to enjoy the festivities.
And then you might just want to tuck a copy into someone's stocking or gift bag, as well.

1. What is the writer's suggestion for the holiday season?


A. Left your life for an apartment in Central Park.
B. Put aside your chores and read the novel.
C. Tuck a stocking into someone's gift bag.
D. Stole a gold ornament and he deceitful.
E. Have fun and solve some puzzles.

Pembahasan: Jawaban bisa dilihat pada paragraph terakhir yaitu B. Put aside your
chores and read the novel.

2. What intrigues the writer about All Through the Night?


A. lt made her curl up on couch all day.
B. It made her ready for any festivities during holiday.
C. She didn’t know the melody of the song in the book.
D. She wanted to escape every time she read the book.
E. She closed it and got relax.

Pembahasan: Intrigues berarti membuat penasaran. Hal yang membuat penasaran dari
reviewer bisa dilihat pada paragraph kedua. Jawaban yang tepat adalah C. She didn’t
know the melody of the song in the book.

LATIHAN SOAL REVIEW TEXT

Breaking and Entering is an unusual film. Will and Sandy are two architects
working on a major urban renewal project in the King's Cross section in London. The area
is unsafe and ready for such a project. Will and Sandy move their offices into a nearby
vacant warehouse.
Will has to deal with constant burglaries at his new office. One night, he saw Miro
trying to break into the building. He chased Miro to his run down apartment block and
watched him return home to his mother, Amira, a Bosnian immigrant who makes a living
tailoring clothes. Soon, Will 'meet' Amira and they begin an affair. And Amira learns that
Will holds the key of her son's future. How far will this mother go to protect her son?
Breaking and Entering is a very interesting film, it is almost entirely character driven. Tis is
not a bad thing although in the film we are exposed to a story or action driven.
Anthony Minghella, the director, creates some of the most believable, interesting
characters these actors have ever played. It is almost painful to watch them on their
journey. Each of the characters makes decisions affecting how their lives will play out, or
change and these decisions and actions affect the story. The characters aren't reacting to
the story. They are changing it.
1. The text is mainly about ....
A. a review of a new film, Breaking and Entering.
B. the process in making the new film, Breaking and Entering.
C. an amusing story dealing with experi-ence in different ways.
D. an account of an unusual or amusing incident retold by the film.
E. the description how the film is accom-plished through a series of steps

2. "Will and Sandy move their offices into a nearby vacant warehouse." (Paragraph 1)
The antonym of the underlined word is ....
a. empty

57
b. fulfilled
c. covered
d. occupied
e. luxurious

3. Which of the following statements is NOT TRUE according to the text?


a. Miro is one of the burglars who tried to break into Will's new office.
b. Will has to deal with constant burglaries at his new office.
c. Miro is Amira's son, a Bosnian immi-grant who makes living by tailoring clothes.
d. The area where the two architects work is unsafe and ready for a renewal pro-
ject.
e. Minghella, the critic, creates some of the most interesting character the actors
have ever played

Menentukan gambaran umum atau informasi tertentu/rinci/tersirat atau rujukan kata atau
makna kata/frasa dari teks tertulis berbentuk Discussion.

7. DISCUSSION TEXT

Discussion is a text which present a problematic discourse. This problem will be


discussed from different viewpoints. Discussion is commonly found in philosophical,
historic, and social text.
Generic Structure of Discussion
 Statement of issue; stating the issue which is to discussed
 List of supporting points; presenting the point in in supporting the presented
issue
 List of contrastive point; presenting other points which disagree to the
supporting point
 Recommendation; stating the writer' recommendation of the discourse
Language Feature of Discussion
 Introducing category or generic participant
 Using thinking verb; feel, hope, believe, etc
 Using additive, contrastive, and causal connection; similarly, on the hand,
however, etc
 Using modalities; must, should, could, may, etc
 Using adverbial of manner; deliberately, hopefully, etc

Examples and structures of the text

Homework
Issue I have been wondering if homework is necessary.
Statement of issue I think we should have homework because it helps us to learn
and Preview and revise our work.
Homework helps people who aren’t very smart to remember what
they have learned. Homework is really good because it helps with
our education.
Statement of But, many times, doing homework is not a great idea. I think we
various viewpoints shouldn’t have homework because I like to go out after school to
a restaurant or the movies. Sometimes homework is boring and
not important.
I think homework is bad because I like to play and discuss things
with my family.

58
A. Example of Discussion Text on Nuclear Power
The government has just published a report which suggest that television is partly
responsible for the serious increase in crime over the last ten years. The exposure of
violence or pornography harmfully effects on children.
Many people who are alive today know what it is like to live in a world without
television. Television as we know is only about forty years old. Yet it is so much a part of
our lives that it seems as if it had always existed.
Some people think that the years before the invention of television were a better
time. They claim that families talked more and did more things together. More books were
read. People used their imaginations more fully. People got more outdoor exercises.
But others disagree. They claim that television is a powerful educational tool. It
informs us of what is going on in the world, from a famine in Africa to a local politics and
fashion. It helps us understand how people live, work, and struggle.
In 1961, Newton Minow, a government official, called prime-time schedules “ a vast
wasteland.” Television is credited with being a great teacher, but it is also blamed for the
poor reading and writing skills of our population. Television gets praised for helping us
understand the people of the world. But it has been accused of helping to destroy family
life. Television keeps us informed about the political issues of the day.
Experts will probably continue to argue about television’s value. But everyone agrees
that it is one of the most significant inventions of the twentieth century.

Contoh soal dan pembahasan


1. The writer wants to tell …..
a. the development of television
b. the advantages of television program
c. how television destroys people’s life
d. how television improves people’s knowledge
e. the influence of television on people’s daily life

Pembahasan: jawaban dari pertanyaan ini bisa dilihat pada paragraph pertama teks
diskusi membahas tentang issue yang akan dibahas. Yaitu e. the influence of television
on people’s daily life

2. Many people claim that television is a powerful educational tool. From this statement
we know that they ….. with the existence of television.
a. Love d. satisfy
b. Agree e. choose
c. Prefer

Pembahasan: didalam teks discussion merupakan gagasan yang mendukung dan tidak
mendukung. Kalimat diatas merupakan salah satu bentuk dukungan terhadap kehadiran
televisi. Jawaban yang tepat adalah b. agree

3. Which of the following is good for children in watching TV?


a. The children should watch the violence on TV
b. Children may watch TV whenever they like
c. There shouldn’t be government censorship of TV program
d. Children should spend all their time to watch TV program
e. Children should be accompanied by their parents in watching TV
Pembahasan: untuk menjawab pertanyaan diatas, carilah bentuk pernyataan yang
positive. Jawaban A sampai dengan D merupakan pernyataan yang kurang baik.
Sehingga jawaban yang tepat adalah E.

59
LATIHAN SOAL DISCUSSION TEXT
Balancing High School and Part-Time Work

You have to consider a number of factors when deciding whether or not to get a
job. The important thing is to arm yourself with as much information as possible, so you
can figure out what choice makes the most sense for you.
If you are considering working part-time, schedule a meeting with your school
counselor to discuss this move. Talk to your counselor about why you want to work and
what type of position you’re seeking. Simply explaining your goals to someone else can
help you make decisions and figure out your priorities.
Schoolwork, including homework and studying for tests, should always be your
top priority. “The activities and courses students choose vary considerably, so it’s
important for young people to keep their individual situations clearly in mind,” says Brad
McGowan, director of the Career Centre at Newton North High School in Massachusetts.
Further, McGowan cautions students who do decide to work: “If you are rushing through
your assignment or not studying enough for tests because of work, it’s time to cut back or
quit and find a less time-consuming job.” He adds, “Students should always let their
employers know what their time limits are.” If you are being pressured to work more hours
than you can handle, you need to find a new place to work. You also need to make sure
that a job won’t prevent you from getting enough rest.
Adapted from: http://www.collegeboard.com

1. What should a student do first when considering working part time?


a. Schedule a meeting with the employer of the job.
b. Discuss with the school counselor.
c. Schedule a meeting with the school principal.
d. Discuss with your friends.
e. Schedule a meeting and discuss with the class.

2. What does Brad McGowan do?


a. Brad McGowan is a student who takes part time job.
b. Brad McGowan is the director of the Career Centre at Newton North High
School.
c. Brad McGowan is the school counselor of Newton North High School in
Massachusetts.
d. Brad McGowan is the employer of some students at Newton North High
School.
b. Brad McGowan is the principal at Newton North High School in
Massachusetts.

3. What does the word “it” in paragraph 3 refer to?


a. student
b. individual situation
c. young people
d. to keep to individual situations clearly in mind
b. to keep to individual situations clearly in mind is important.

60
4. What does Brad McGowan suggest when students don’t have enough time to
study?
a. Quit and find a less time-consuming job.
b. Talk to the employer.
c. Discuss with the school counselor.
d. Keep doing both working while studying.
e. Sue the employer for the compensation.

LATIHAN SOAL UN 2017 DISCUSSION TEXT


47 This text is for questions 47 to 50. Disajikan teks
tulis berbentuk
Nowadays, the police have been applying the new
discussion
regulation concerning the use of seat belts. In European
peserta didik
countries, this regulation has been applied for a long time.
dapat
However, this new regulation has become controversial
menentukan
and is an interesting topic to discuss. Here are some of the
pernyataan yang
arguments.
tepat terkait
The use of seat belts has been proven to reduce the risk of dengan
injury or death in an accident. Seat belt has become a tujuannnya
standard component in cars. The research shows that most
car accident will cause an injury to the head. Frequently,
drivers or the passengers driving without seat belts die
because of this. By wearing the seat belts, the injury will not
happen since belts restrain our body on the car seat when
the accident happens.
Unfortunately, many cars, especially the old ones, don’t
have seat belts. This is because the traffic conditions in the
past were unlike the recent traffic conditions. The designer
of old cars didn’t consider a seat belt as an important part.
Besides, the drivers wearing the seat belt will think that
they are completely safe, so they may drive carelessly.
They are safe, indeed, but how about safety of other? The
seat belt is only one of the ways to reduce the risk of car
accidents. It doesn’t mean that we are completely safe. In
short, our safety depends on ourselves.

47. What is the main idea of paragraph 3?


A. Old cars do not use seat belts.
B. The old cars have seat belts for their safety.
C. The drivers wear seat belts may drive carefully.
D. The traffic conditions in the past were similar to the
recent.
E. The old cars’ designer considered a seat belt as an
important part.

48 48. What is the function of seat belts? Disajikan teks


tulis berbentuk
A. To reduce the traffic conditions.
discussion
B. To become a standard component in cars. peserta didik
dapat
C. To reduce the risk of injury or death in an accident. menentukan

61
D. To prevent the drivers driving cars carefully. pernyataan yang
tepat terkait
E. To increase the risk of injury in car accidents.
dengan jenis-
jenis informasi
yang tercakup
dalam perincian
tentang
obyeknya

49 49. Who considered a seat belt as an unimportant part? Disajikan teks


tulis berbentuk
A. Police.
discussion
B. People. peserta didik
dapat
C. Drivers. menentukan
D. Passengers. pernyataan yang
tepat
E. Old cars designer.

50 50. “... that they are completely safe.” (Paragraph 3) What Disajikan teks
does the word ‘they’ refer to? tulis berbentuk
discussion
A. Cars.
peserta didik
B. Seat belts. dapat
menentukan
C. Drivers. kata/frasa/klausa
D. Designers. yang tepat yang
dirujuk oleh kata
E. Others. ganti yang
digarisbawahi
pada kalimat
dari teks

Menentukan gambaran umum atau informasi tertentu/rinci/tersirat atau rujukan kata atau
makna kata/frasa dari teks tertulis berbentuk Report Text.

8. REPORT TEXT
Report is a text which presents information about something, as it is. It is as a
result of systematic observation and analysis.
Generic Structure of Report
1. General classification: Stating classification of general aspect of thing; animal,
public place, plant, etc which will be discussed in general
2. Description: Describing the thing which will be discussed in detail; part per part
, customs or deed for living creature and usage for materials

Language Feature of Report


• Introducing group or general aspect
• Using conditional logical connection; when, so, etc
• Using simple present tense

62
Examples and structures of the text
THE PELICAN REPORT
General The white pelican is one of the most successful fish-eating
Clasification birds.
Description The success is largely due to its command hunting behaviour.
A group, perhaps two dozen birds, will gather in a curved arc some
distance offshore. The birds then begin to move forward towards the
shore, beating the water furiously with their wings, driving the fish
before them.
When the water is shallow enough for the birds to reach the
fish, the formation breaks up as each bird dips its bill into the water to
scoop up its meal. As the bird lifts its head, the water drains from its
bill leaving the fish which are then swallowed.
Pelicans are among the oldest group of birds, Fossils of this
genus have been found dating back 40 million years.

Contoh soal dan pembahasan

A cauliflower is a plant of the cabbage family, Brassicaceae. There are four main
groups of cauliflower. They are Italian, Northern European annual, Northwest European
biennial, and Asian. Italian cauliflowers have various colors, such as green, white, purple
and yellow. Northen European annual cauliflowers are grown in Europe (especially
France). They are harvested in spring and summer. Asian cauliflower are grown in India
and China.

1. What does the text talk about?

a. What cauliflowers contain d. The price of cauliflowers

b. How to grow cauliflowers e. The kinds of cauliflowers

c. The benefit of cauliflowers

Pembahasan: Teks report diatas membahas kembang kol. Digambarkan bahwa kembang
kol mempunyai beberapa jenis. Oleh karena itu jawaban yang tepat adalah E.

2. Which cauliflowers are grown in France?

a. Northwest European biennial cauliflowers

b. Northern European cauliflowers

c. Purple cauliflowers

d. Italian cauliflowers

e. Asian cauliflowers

Pembahasan: Pertanyaan ini merupakan informasi spesifik didalam teks. Didalam teks
dinyatakan bahwa jenis kembang kol yang tumbuh di Eropa terutama di Perancis adalah
jenis Northwest European biennial. Jawaban yang tepat adalah A.

3. Which cauliflowers can we find in Italy?

a. Sprouting Broccoli d. Winter cauliflowers

b. Purple cauliflowers e. Semi-Asian cauliflowers

c. Spring cauliflowers

63
Pembahasan: pertanyaan ini menanyakan informasi tersirat dalam teks. Didalam teks
dinyatakan bahwa kembang kol yang tumbuh di Itali mempunyai warna yang beragam,
diantaranya Purple, green, white dan yellow. Karena itu jawaban yang tepat adalah B.

LATIHAN SOAL UN 2017 REPORT


27 Questions 27 to 30. Disajikan teks tulis ilmiah
faktual (factual report)
sederhana tentang benda
A kangaroo is an animal found only in Australia. It has /binatang / gejala alam/
a small relative called a wallaby, that lives in peristiwa alam dan sosial,
Tasmania and New Guinea. Kangaroos eat grass and peserta didik dapat
plants. They have short front legs, long and strong menentukan pernyataan yang
back legs and a tail. These are used to sit up and tepat terkait tujuannya
jump. (PURPOSE)

Kangaroos are well known for their 8-meter forward


jumps and more than 3-meters high leap across
fences. They can also run at the speed of over 45
kilometres per hour. The largest kangaroos are the
Great Grey Kangaroos and Red Kangaroos. Adult
kangaroos grow to a meter ¡n length and 90 kilos in
weight. Kangaroos are marsupials. This means that
the female kangaroo has an external pouch on the
front of her body. The baby kangaroo is very tiny
when it was born. It right away crawls into its mother’s
pouch where it spends the first five months of its life.
Adapted from Peter Haddock Ltd, Ref.O8J

27. What is the purpose of the text?


A. To inform Kangaroo’s uniqueness.
B. To report the features of Kangaroo.
C. To encourage Kangaroo’s preservation.
D. To describe how a Kangaroo looks like.
E. To raise awareness of Kangaroo’s special status.

28 28. We can conclude from the text that.... Disajikan teks tulis ilmiah
faktual (factual report)
A. the largest kangaroos are the males
sederhana yang sama
R. the male kangaroos do not have pouch tentang benda /binatang /
gejala alam/ peristiwa alam
C. kangaroos only eat grass and plants that grow in dan sosial, peserta didik
Australia dapat menentukan
D. Tasmania’s and New Guinea’s wallabies are pernyataan yang tepat terkait
different in size dengan jenis-jenis informasi
yng tercakup dalam perincian
E. the new born kangaroo is also skilful in jumping tentang obyeknya
and leaping

29 29. Which of these statements is contrary to the fact? Disajikan teks tulis ilmiah
faktual (factual report)
A. Kangaroos are not carnivorous.
sederhana yang sama
B. Kangaroo is native animal of’ Australia. tentang benda /binatang /
gejala alam/ peristiwa alam
C. All wallabies are of smaller size than Kangaroos. dan sosial, peserta didik
dapat menentukan

64
D. All marsupial females have pouch on the front of pernyataan yang tepat terkait
their body. dengan deskripsi obyeknya
E. Baby Kangaroos leave their mother’s pouch at the
age of’ five months.

30 30. Look at the underlined word ‘pouch’. Which of the Disajikan teks tulis ilmiah
following words is closest in meaning to it? faktual (factual report)
sederhana yang sama
A. Saving.
tentang benda /binatang /
B. Purse. gejala alam/ peristiwa alam
dan sosial, peserta didik
C. Compartment. dapat menentukan kata yang
D. Pocket tepat yang memiliki
kesetaraan makana dengan
E. Holding. kata yang digarisbawahi pada
kalimat dari teks.

9. PROCEDURE TEXT

Text prosedur bertujuan untuk memberi petunjuk tentang langkah-langkah/metode/cara-


cara melakukan sesuatu. Teks prosedur umumnya berisi tips atau serangkaian tindakan
atau langkah membuat suatu barang atau melakukan suatu aktivitas. Teks ini dikenal juga
dengan istilah directory. Teks procedure umumnya memiliki struktur:

4. Goal; tujuan kegiatan

5. Materials; bahan – bahan yang dibutuhkan untuk membuat barang atau melakukan
suatu aktivitas

6. Steps; serangkaian langkah

Teks prosedur sering menggunakan unsur kebahasaan tertentu, antara lain:

1. Imperative; kalimat – kalimat perintah, misalnya go, sit, don’t put, don’t mix, dsb.

2. Action Verb; kata kerja yang berhubungan dengan aktivitas fisik atau intelktual,
misalnya mix, turn, don’t put, dsb.

3. Connective of sequence; misalnya, then, while, next, dll.

4. Numbering; angka-angka yang menunjukkan urutan kegiatan, misalnya first, second,


third, dll.

Contoh Soal dan Pembahasan

2. Find a safe place, uncrowded spot on the beach. The water should not be too choppy
so that will get a clean ride.

3. Use a light, small, fiberglass board with a leg rope and a wet suit if it’s cold.

4. If you are more experienced, you could try kneeling on the board once you are on the
wave.

5. The most important thing is to keep your balance or else you will end up falling off the
board.

6. Wait untill you see a small wave then lie on your surfboard. When the wave is close,
start paddling furiously.

7. Don’t go out too far if you haven’t surfed before.

65
Q: The text mainly tells us about....

i. Warning of the danger to surf

ii. Tips to surf carefully

iii. Things to have prior to surfing

iv. Instruction for a beginner surfer

v. How to overcome surfing danger

Pembahasan:

Secara keseluruhan teks memaparkan cara – cara untuk berselancar dengan aman, baik
untuk pemula maupun yang sudah proffesional. Jawaban yang tepat B.

Q: the word choppy in step 1 closely means...

A. Uneven

B. Dirty

C. Fine

D. Smooth

E. Streamlines

Pembahasan:

Choppy diartikan berombak. Dari semua pilihan yang paling mendekati adalah pilihan
eneven (tidak rata). Jawaban A.

LATIHAN SOAL UN 2017

23 The following text is for questions 23 to 26. Disajikan teks tulis prosedur
berbentuk manual atau kiat-kiat
(tips), peserta didik dapat
If someone is having a nose bleed, your priority is to menentukan pernyataan yang
control the bleeding and keep their airway open. tepat terkait dengan tujuannnya.

Get them to sit down (not lie down) as keeping the


nose above the heart will reduce bleeding.
Get them to lean forward (not backwards), to make
sure the blood drains out through their pause every
ten minutes, until the bleeding stops.
Encourage them not to speak, swallow, cough, spit
or sniff because this may break blood clots that may
have started to form in the nose.
If the bleeding is severe, or if it lasts more than 30
minutes, call for medical help.

23. What is the purpose of the text?


A. To inform the readers the methods to help nose
bleeding patients.
B. To report the procedural steps in helping nose
bleeding patients.
C. To illustrate the preventive methods of nose

66
bleeding.
D. To provide instant help for nose bleeding
patients.
E. To describe the control methods of nose
bleeding.

24 24. The text is mostly useful in a condition .... Disajikan teks tulis prosedur
berbentuk manual atau kiat-kiat
A. when the bleeding cannot he controlled
(tips), peserta didik dapat
B. when the patient’s airway is blocked menentukan pernyataan terkait
dengan langkah tindakan di
C. when the nose bleed occurs regularly dalamnya
D. when the nose bleed just occurred
E. when the patients keep coughing

25 25. In case of nose bleeding, if the person speaks, . Disajikan teks tulis prosedur
... berbentuk manual atau kiat-kiat
(tips), peserta didik dapat
A. his/her blood drains out though his/her nose
menentukan pernyataan terkait
B. he/she will suffer from severe bleeding dengan langkah tindakan di
dalamnya
C. his/her throat will block the airway
D. his/her blood clots will not form
E. his/her nose bleeds again

26 “ … to make sure the blood drains out …” Disajikan teks tulis prosedur
(paragraph 3) berbentuk manual atau kiat-kiat
(tips), peserta didik dapat
The underlined word is closest in meaning to …
menentukan pernyataan terkait
A. travel dengan langkah tindakan di
B. portray dalamnya
C. remark
D. stream
E. proceed

67
LATIHAN SOAL READING TEXT

In total, the plant has need of at least 16 elements, of which the most important are
carbon, hydrogen, oxygen, nitrogen, phosphorus, sulfur, potassium, calcium, and
magnesium.
The plan obtains carbon and hydrogen dioxide from the atmosphere; other nutrients are
taken up from the soil. Although the plant contains sodium, iodine, and cobalt, these are
apparently not essential. This is also true of silicon and alumunium.
Overall chemical analyses indicate that the total supply of nutrients in soils is
usually high in comparison with the requirements of crop plants. Much of this potential
supply, however, is bound tightly in forms that are not released to crops fast enough to
give satisfactory growth. Because of this, the farmer is interested in measuring the
available nutrient supply as contrasted to the total quantities

1. Which of the following elements is not taken up from the soil?


A. Potassium D. Calcium
B. Magnesium E. Sulfur
C. Carbon

2. Why do farmers prefer considering the only available nutrients to the total
quantities of nutrient found in the soil? Because ...
A. they do to know how to obtain all the nutrients
B. chemical analyses indicate that the soil constrains all the nutrients needed
C. the nutrient are in the soil
D.most of the nutrients are bound in compounds
E. some of the nutrients are found in the atmosphere

3. The topic of paragraph three is about ...


A. the result of chemical analyses
B. the requirements of crop plants
C. giving satisfactory growth to the plants
D. measuring the available nutrients supply
E. the abundant of nutrients in the soil

4. "The plant obtains carbon and hydrogen dioxide ..." (paragraph 2). Obtains
means...
A. gets
B. takes
C. puts
D. gives
E. has

Read the following text and answer number 6-8!

Once upon a time, a rabbit wanted to cross a river but he could not swim. He had an idea.
He saw a boss of crocodile swimming in the river. The rabbit asked the boss of crocodile,
"How many crocodiles are there in the river?" The boss of crocodile answered, "We are

68
twenty here." "Where are they?" the rabbit asked for the second time. "What is it for?" the
boss of crocodile asked.

"All of you are good, nice, gentle and kind, so I want to make a line in order. Later I will
know how kind you are," said the rabbit. Then, the boss of the crocodile called all his
friends and asked them to make a line in order from one side to the other side of the river.
Just then, the rabbit started to count while jumping from one crocodile to another: one ...
two ... three ... four ... until twenty, and finally, he thanked all crocodiles because he had
crossed the river.

5. The story mainly tells us about ...


A. twenty crocodiles
B. the boss of the crocodile
C. a rabbit and twenty crocodiles
D. a rabbit and the boss of crocodile
E. the boss of the crocodile and ail his friends

6. We know from the first paragraph that the rabbit actually wanted ...
A. to cross the river
B. to swim across the river
C. to meet the boss of crocodile
D. to know where the crocodiles are
E. to know the number of crocodiles there

7. All of you are good, nice, gentle, and kind ..." (Paragraph 2)
The underlined word is synonymous with ...
A. wild
B. diligent
C. cheerful
D. easygoing
E. honorable

This text is for questions 8.


Hello my friend, how are you doing, there? I hope you are in good condition. I am sending
this letter to give you some good news.

It's been one year we separated, since you moved to Makassar. I miss you so much. I'm
really looking forward to meeting with you soon. After waiting for so long, I’m finally
granted permission to visit you in Makassar. Maybe next month I will go there by plane.
When I got there, you have to take me around to the tourist attractions there. I really
wanted to visit the Losari beach. See you there buddy.

Your friend,

Penny

69
8. I’m finally granted permission to visit you in Makassar. What is the closest
meaning of the underlined word?
a. Answered
b. Allowed
c. Fullfilled
d. Accepted
e. fixed

Text for question 9 - 11


An elephant is the largest and strongest of all animals. It is a strange looking
animal with its thick legs, huge sides and backs, large hanging ears, a small tail, little eyes,
long white tusks and above all it has a long nose, the trunk.
The trunk is the elephant's peculiar feature, and it has various uses. The elephant
draws up water by its trunk and can squirt it all over its body like a shower bath. It can also
lift leaves and puts them, into its mouth. In fact the trunk serves the elephant as a long arm
and hand. An elephant looks very-clumsy and heavy and yet it can move very quickly.
The elephant is a very intelligent animal. Its intelligence combined with its great strength
makes it a very useful servant to man and it can be trained to serve in various ways such
as carry heavy loads, hunt for tigers and even fight.

9. The third paragraph is mainly about the fact that ...


A. elephants are strong
B. elephants can lift logs
C. elephants are servants
D. elephants are very useful
E. elephants must be trained

10. Which of the following is NOT part of the elephant described in the first paragraph?
A. It looks strange.
B. It is heavy.
C. It is wild.
D. It has a trunk.
E. It has a small tail.

11. "The trunk is the elephant's peculiar feature ..." (Paragraph 2)


The underlined word is close in meaning to ...
A. large
B. strange
C. tough
D. smooth
E. long

70
Text for question 12 - 14

Dear, Rosa

I am a guy, twenty-eight years old. I have a problem with a girl friend. She is twenty five
years old. We have been dating each other for years, since we were high school students.
Now we have jobs. I feel very sure that we love each other. Now it is time for me to get
married. Unfortunately, her parents never approve of our relationship ever since the very
first time. My family, anyhow, does not really mind. At first I thought my girl friend had the
power to defend our love. But then she surrendered. She loves, me and she loves her
family, too. On the one hand she said, "I'll be happy if my family is happy." It means she
had to get approval from her parents.

For this I was shocked. My heart was broken. She ended our relationship just at the time
when I was ready to marry her. When I was with her, we were very happy. We had a lot in
common and there were no conflicts. The truth was, we separated only because of her
parents. They want her to marry her cousin, her aunt's son (this is one of the traditions in
the Karonese).

For the time being I am very upset. When I miss her I call her. Then we can eat together,
talk together for hours. We act like elating because she loves me, too. The real problem is,
she cannot refuse what her parents want. So, should I forget my girl friend, the only one
that I truly love? Or else, should I just wait, and dream that one day she will be back to
me?
Rio N., Padang

Dear Rio,

You call her your girl friend, but she has chosen her own way. For this reason I’d, say that
there is nothing you can do except forget her and go on with your normal life. Hopefully
you can find another and livehappily forever.
Rossa

12. The main idea of the first paragraph is that ...


A. the writer wants to get married
B. the writer has been dating for years
C. the writer has a problem with his girl friend
D. the writer feels sure that they love each other
E. the writer's parents disapprove of his relationship

13. Which of the following sentences is TRUE according to the text?


A. Rio is not ready to get married.
B. Rio's parents disagree to his marriage.

71
C. Rio and his girl friend often got conflicts.
D. Rio's girl friend didn't really love Rio.
E. Rio's girl friend couldn't defend her love for Rio.

14. Why do the girl's parents disapprove of their marriage? Because ...
A. they don't like Rio.
B. the girl is still studying.
C. Rio is from a different ethnic group.
D. they keep the Karonese tradition.
E. the girl has chosen another guy.

Text for question number 15-17.

Two students were discussing the school's new rule that all the students must
wear a cap and a tie. One of them showed her annoyance. She said that wearing a cap
and a tie was only suitable for a flag rising ceremony. So, she was against the rule.
Contrary to the girl's opinion, the other student was glad with it. He said that he didn't
mind with the new rule because wearing a cap and a tie will make the students look great
and like real educated persons. The first student gave the reasons that they would feel
uncomfortable and hot. Moreover, the classrooms were not air conditioned. The second
said it wasn't a big problem. He was sure that the students would wear them proudly.
They would surely be used to it any way.

15. The two students are discussing ...


A. the facilities in school
B. their homework
C. their uniform
D. their friends
E. their family

16. The boy said that he agreed with the new rule in his school.
Which statement shows his agreement?
A. He was not annoyed.
B. He would not obey the rule.
C. He didn't care of the rule.
D. He didn't like wearing a cap and tie.
E. He didn't mind wearing a cap and tie.

17. The boy believed that all students would ...


A. have a high spirit to study
B. solve their own problems
C. care for their environment
D. follow the new rule
E. feel uncomfortable

72
This text is for items number 18-23

Many people now have a card which enables them to withdraw money from
a cash dispenser. You feed your card into the machine and key in your PIN
(Personnel Identification Number) and the amount of money you want If you have
enough in your account, the money requested will be issued to you up to a dairy
limit. Your account is automatically debited for the amount your have drawn out.
Provided you have a sound credit, you can get a credit card from a bank and other
financial institutions. To obtain goods or services, you present your card and sign a
special voucher. When it receives the voucher, the credit card company pays the
trader (less a commission) and then sends you a monthly statement. Depending on
the type of card you have, you will either have to pay in full or be able to pay part of
what is owed and pay interest on the balance left outstanding.
If you need to make fixed payments at regular intervals, e.g. for insurance
premiums, you can arrange a standing order (sometimes known as a banker's
order) so that the bank will do this for you.

18. If you withdraw your money from the cash dispenser the amount of your money will
... as you have drawn out.
A. become more
B. be credited
C. become lost
D. become less
E. be doubled

19. The main purpose of the writer is to ...


A. give the reader information about bank service
B. persuade die readers to have a credit card
C. check the customers' account
D. explain to the reader how to use PIN
E. explain to the customers how to pay their employee's salaries

20. Which of the following is the main idea of the second paragraph?
A. The credit card company pays the traders.
B. The credit card company sends you a monthly statement.
C. Banks and other financial institutions offer us credit cards to get goods and
services.
D. Other financial institutions help the banks to provide credit cards.
E. A voucher is the only way to buy goods.
21. "Many people BOW have a card which enables them ______.." (Paragraph. 1).
"enables" means ...
A. forces
B. supports

73
C. asks
D. allows
E. encourages
22. If you have enough money in your account ...
A. you can withdraw your money from a cash dispenser.
B. you have a special voucher.
C. the bank sends you a monthly statement.
D. you have paid special interest.
E. the type of card is acceptable.

23. Mrs. Anwar : I heard that your son had won a scholarship to Pajajaran University.
Mrs. Budiman : Yes, that's right .... He never ignores our advice and always studies
hard.
A. I'm proud of him
B. I prided myself on his success
C. His pride would not let him down
D. I'm too proud to have been successful
E. He is a profile of a successful student

This text is for questions 24 to 27.

University of Cambridge

Do you plan to study abroad? Don't hesitate. Welcome to Cambridge University.


Cambridge University, an institution of higher education, is the second oldest university in
Great Britain after the University of Oxford. It is located in the city of Cambridge.

The University of Cambridge is a system of faculties, departments, and 31 independent


colleges. You know, although the colleges and the university are separate corporations, all
are parts of an integrated educational entity. The university examines candidates for
degrees during their residencies and at the conclusion of their studies. The colleges
provide their students with lodgings and meals, assign tutors, and offer social, cultural, and
athletic activities. Every student at the University of Cambridge is a member of a college.
Let's see its academic year. The academic year is divided into three terms of
approximately eight weeks each: Michaelmas (autumn), Lent (late winter), and Easter
(spring). Students required to study under supervisor are usually members of the college's
faculties who maintain close relationships with small groups of students in their charge and
assist them in preparing for university exams.

24. The author's purpose of writing the text is ...


A. to review a particular education system
B. to commemorate a particular college
C. to define a particular academic year

74
D. to explain a particular way to study
E. to describe a particular institution

25. Which of the following statements is TRUE?


A. The academic year is held in four seasons in a year.
B. Students in colleges are not members of the university
C. Students must not be in their residence during the terms.
D. The students of Cambridge University have holidays in summer.
E. University of Oxford is younger than University of Cambridge.

26. The second paragraph tells the readers about ...


A. an integrated educational entity.
B. social activities in the university.
C. the system in Cambridge University.
D. the examination for candidates' degrees.
E. the criteria for the membership of the university.

27. "The colleges provide their students with lodgings and meals, assign tutors and
offer ..." (Paragraph 2).
The underlined word means ...
A. dormitories used for studying in groups
B. places offered for doing some business
C. spaces needed for discussion
D. houses needed for taking a rest
E. rooms rented to stay in

Text for questions number 28-29.

We are announcing today that we are bringing the Milestone and Ever Green brands even
closer together. Effective as of December 5, 2005, our official name will be:
GREEN MILES WEST
The substitution of "West" in our name-replacing "California"- is the result of an agreement
we reached with California Gardening Association, following a protest over the original use
of "California" in our name. We hope this does not create any confusion among our loyal
consumers. While this represents a change from our initial name introduction, it does not
change the quality of products we offer to our consumers.

28. Which of the following statements is TRUE according to the text?


A. The corporate offices were protested
B. The loyal consumers created an official name for the company.
C. There was a conflict between Green Miles West and Milestone
D. The quality of products will be different from the former products.
E. The name "Green Miles West" will be effective as of December 5, 2005.

75
29. “… it does not change the quality of the products we offer ..." (Paragraph 4).
The underlined word means ...
A. take D. erase
B. lose erase E. throw
C. alter

Text for question 30.


Bacharuddin Jusuf Habibie is the third President of Indonesia. He was born on
June 25, 1936 in Pare-Pare, South Sulawesi. He is the fourth of eight children in his family.
His parents are Alwi Abdul Jalil Habibie and RA. Tuti Marini Puspowardojo. He spent his
childhood in Pare-Pare. Since he was a kid, he has shown his integrity and persistent
behaviour. At school, Habibie whose hobbies are riding horse and reading book is famous
for his intelligence.
Habibie’s father passed away due to heart attack when he was in elementary
school. After that, his mother sold their vehicle and house and then moved to Bandung. As
a single parent, Habibie’s mother worked hard to afford her life with her children.
In 1954, after Habibie graduated from senior high school in Bandung, he continued
his study to Bandung Institute of Technology. He did not finish his study at ITB because at
the same time he got a scholarship from The Ministry of education and Culture of
Indonesia to study in Germany. He remembered that Bung Karno had ever said about the
importance of plane development for Indonesia, and then he decided to take aerospace
engineering with aeroplane construction specialty at Rhein Westfalen Aachen Technische
Hochschule (RWTH), an institute of technology in Germany.
Habibie studied hard to be success since he remembered how hard his mother
worked for his life. Even, Habibie spent his time in holiday to earn extra money for buying
books. When holiday ended, he did not do anything except studying. In 1960, he
graduated with great score, almost perfect (9.5). With his education background, he
applied at an industry company of train, named Firma Talbot.
After that, he continued his doctoral degree at Technische Hochschule Die
Facultaet Fuer Maschinenwesen Aachean. In 1962, he got married with Hasri Ainun. She
was a doctor and she followed his husband to Germany. Habibie and Ainun has two sons.
They are Ilham Akbar dan Thareq Kemal. At that time, life got harder. He had to go to work
in the early morning because he walked to the office to save more money. He went home
in the evening and studied. He finished his doctoral degree in 1965 with perfect score (10).

30. What can we infer from the third paragraph?


a. Habibie continued studying to ITB
b. Habibie remembered Soekarno’s saying
c. Habibie decided to take aerospace engineering
d. Habibie’s studied hard from senior high school to Institute of technology Germany
e. Habibie did not finish his study in ITB

76
TRYOUT 1

Listening Section
(take the audio here:
ttps://drive.google.com/open?id=0B_QMc2t7MMndTVE3V09WbTBSdDg)

In this section of the test, you will have the chance to show how well you understand
spoken English. There are four parts to this section with special directions for each part.

PART I
Question 1 to 3.
Directions:
In this part of the test, you will hear some dialogues and questions spoken in English. The
dialogues and questions will be spoken twice. They will not be printed in your test book, so
you must listen carefully to understand what the speakers are saying.
After you listen to the dialogue and the question about it, read the five possible answers,
and decide which one would be the best answer to the question you have heard. Now
listen to a sample question.

You will hear:


Boy : Do you use the internet very often?
Girl : Yes, I do. I use it for communication.
Boy : How do you communicate with it?
Girl : I send and receive emails, and I speak with my friends using voice mail.

Narrator: What is the main topic of the conversation?


Sample answer

You will read in your test book:

A. How to use email.


B. Receiving emails.
C. Communication.
D. Internet usage.
E. Voice mail.
The best answer to this question is “Internet usage”. Therefore you should choose answer
(D)

1.
A. Parking area.
B. Looking for parking.
C. Parking the car in the lot.
D. Walking across the street.
E. Taking the car to the mechanic.

2.
A. To copy it.
B. To recheck it.
C. To throw it away.
D. To show it to Mrs. Hiroko.
E. To submit it to the director.

3.
A. Meat.
B. Butter.
C. Lettuce.
D. Tomato.
E. Mustard.

77
PART II

Questions: 4 to 8.

Directions:

In this part of the test, you will hear some incomplete dialogues spoken in English,
followed by five responses, also spoken in English. The dialogues and the responses will
be spoken twice. They will not be printed in your text book, so you must listen carefully to
understand what the speakers are saying. You have to choose the best response to each
question.

Now listen to a sample question.

You will hear:

Man : Where are we going to stay?

Woman : At the Sun Hotel, near the beach.

Man : Why there?

Woman : ….

Narrator: What will the woman most likely reply?

Sample answer

You will also hear:

A. Staying there.
B. It’s on the beach.
C. Living in the hotel.
D. There’s a hotel there.
E. It provides excellent service.

The best answer to the question is choice E “It provides excellent service.” Therefore, you
should choose answer E.

4. Mark your answer on your answer sheet.


5. Mark your answer on your answer sheet.
6. Mark your answer on your answer sheet.
7. Mark your answer on your answer sheet.
8. Mark your answer on your answer sheet.

PART III

Question: 9 to 11.

Directions:

In this part of the test, you will hear some dialogues and a monologue spoken in English.
The dialogues and monologues will be spoken twice. They will not be printed in your text
book, so you must listen carefully to understand what the speakers are saying. After you
listen to a dialogue or monologue, look at the five pictures provided in your test book, and
decide which one would be most suitable with the dialogue or monologue you have heard.

78
9.

1 2

3 4

A. 1
B. 2
C. 3
D. 4
E. 5
10.

1 2

79
3 4

A. 1
B. 2
C. 3
D. 4
E. 5
11.

1 2

3 4

80
A. 1
B. 2
C. 3
D. 4
E. 5

PART IV

Question 12 to 15.

Directions:

In this part of the test, you will hear several monologues. Each monologue will be spoken
twice. They will not be printed in your text book, so you must listen carefully to understand
what the speakers are saying.

After you hear the monologue and the question about it, read the five possible answers
and decide which one would be the best answer to the question you have heard.

12.
A. Tropical regions.
B. Sunshine.
C. Bananas.
D. Rain.
E. Fruit.

13.
A. In the world.
B. In the regions.
C. In the high trees.
D. In the small root.
E. In tropical regions.

14.
A. How to get the gift.
B. How to make a doll.
C. How to make gift-wrap paper.
D. What boxless candies or chocolate is.
E. How to wrap a doll or some boxless candies.

15.
A. Dipping the bottom.
B. Stampping the crepe paper.
C. Spreading some crepe papers.
D. Cutting a piece of crepe paper.
E. Waiting until the bleach is dry.

This is the end of the listening section

Reading Section

81
The following text is for questions 16 and 17.

To : All teachers

From: Wilma Harris, Principal.

Re : Attendance forms

It has come to my attention that many of you have been turning in your attendance
forms late. Please remember that the deadline for submitting attendance forms is 3.30
P.M. on the last school day of every month. Also, you call the parents of every child
who is absent three or more times without an excuse.

Thank you.

16. Why did Wilma Harris write this message?


A. Teachers have been submitting their attendance forms late.
B. Too many students have been absent from school.
C. Students have been paying attention in class.
D. Teachers have been arriving at school late.
E. Attendance forms are empty.

17. You must call the parents of every child who is absent three or more times without an
excuse.
The synonym of the underlined word is ….

A. opinion
B. regret
C. ignore
D. reason
E. propose

The following text is for questions 18 to 19.


Dear Siti Fatimah,

How are you? We have separated for almost two years and we have never met each
other since then. You know, I am busy with my work in Jakarta and you are in Yogyakarta. Oh, I
really miss you.

Siti Fatimah, I want to tell you about my holidays. Last week I visited Bandung, a place
where we spent our childhood. I stayed there for a week. Arriving there, past memories came into
my mind. I remembered our togetherness. I contacted our old friends at Senior High School and
we met in our favorite park. It was such a small but nice reunion party, you know. I really wished
that you could join us. Did you know that Shanty lives in Bali now?

I think that’s what I want to share with you. Always keep in touch, okay?

Love,

Choirunnissa

18. The letter is about....


A. Choirunnissa’s holidays
B. Choirunnissa’s old friends
C. Choirunnissa’s job in Jakarta
D. Choirunnissa’s reunion party
E. Choirunnissa’s past memories

82
19. Choirunnissa writes the letter to....
A. tell Siti Fatimah about her childhood
B. give information about her arrival
C. invite Siti Fatimah to join her to go with her
D. tell Siti Fatimah about her activities in her last vacancy
E. ask for information about when she and Siti Fatimah can meet

The following text is for questions 20 and 21

TENDER ANNOUNCEMENT

Hess (Indonesia-Pangkah) Limited, a Production Sharing Contractor of BPMIGAS, invites Service


Providers to participate for the following tender:

Title Ref.No. Local Content

Provision of Housing BID-11D0044 35%


Maintenance: for 2014-2015
Period

Registration: should you wish to participate, you may collect the Pre-qualification Form at the
schedule given bellow:

Date : March 6-9, 2014

Time : 9 a.m. – 3.30 p.m.

Place : Hess (Indonesia-Pangkah) Limited. The energy of 11th floor, SCBD LOT 11A

Jl. M.T.
20. The Haryono
above Kavc.52-53,isJakarta
announcement 12190
an offer to ….
A. BPMIGAS
Jakarta, 6 March 2014
B. Hess Limited
C. Bid Committee Hess (Indonesia-Pangkah) Limited
D. Service Provider
Bid Committee
E. Provision of housing

21. “Registration: …, you may collect the …”


The underlined word is synonymous with …

A. put
B. get
C. raise
D. spring
E. distribute

83
The following text is for questions 22 and 23

The Philippines’ Best

You Simply Don’t Want to Leave

By Michael D Marasigan

They may be separated by political differences but the 7,107 islands of the Philippines
are united by hospitality of its people. Getting in could be very easy but once you’re in
the Philippines, getting out is more diffficult. You simply don’t want to leave.

It may be hard to believe but it’s true, especially if you’ve found your paradise in one of
its islands. This is exactly why the Philippines, a nation of more than 80 million people, is
a mixture of so many cultures – Asian, European, American, and many more.

22. There are ... islands of the Philippines.


A. seven hundred and seven
B. seven hundred and seventeen
C. seven thousand one hundred and seven
D. seven thousand one hundred and seventy
E. seven thousand one hundred and seventeen

23. We can infer from the ads that....


A. the Philippines people are unfriendly
B. the Philippines consists of 7,707 islands
C. it’s difficult to leave the Philippines once you get in
D. there are less than 80 million people in the Philippines
E. the Philippines country is a mixture of Asia and European

The following text is for questions 24 to 26.

After we finished the semester examination, the students of our school had a trip to
Parang Tritis beach. The students consisted of 20 males and 20 females. All of them were
from Solo.

The trip was held on Sunday, December 13, 2013. We started from Solo at 08.00
o’clock by bus. Parang Tritis is a beach located in Bantul regency, about 45 kilometres
from Jogja. It is about two hours to go there.

It was indeed a pleasant trip. We were smiling, laughing and making jokes. The
scenery along the road was very beautiful. Very large rice fields were spread on both sides
of the road. Here and there we could see beautiful and green mountains among others so
called Pegunungan Seribu. There was also a large sugar cane plantation with its great
sugar factory. We stopped over in the near place, to get some refreshments, some food
and drink.

We arrived at Parang tritis at about 10.00 o’clock. The beach was crowded, because it
was Sunday and holiday. Parang Tritis was a fantastic beach with its white sand. There
were many tourists go there for sunbathing. The breeze always breezed very softly. It gave
us peacefulness to the beach. The waves crumbled as they reached the beach. This went
on again and again so that it gave monotonous rhythm of the scene.

The students, males and females went along the beach to see the scenery of the
Parang Tritis. They could also ride a horse cart if they felt tired. There were also people
who played kites. The students played volley ball. Others stood on the side of the beach.

There were some bungalows along the beach. People who wanted to enjoy the night
view of the beach and stayed the night at the bungalows, got up in the morning to watch

84
the rising sun. Parang Tritis was indeed very beautiful. So when you go to Jogjakarta,
don’t forget to visit the beach.

24. What does the text tell us about?


A. The activities after the semester examination.
B. The description of Parang Tritis beach.
C. The location of Parang tritis beach.
D. The trip to Parang tritis beach.
E. The scenery along the road.

25. What can we conclude from the third paragraph?


A. We could see large rice fields on the slopes of the green mountain.
B. There was a large tea plantation on both side of the road.
C. The view along the street was very wonderful.
D. Pegunungan Seribu was our last destination.
E. We stopped over near the sugar factory.

26. The waves crumbled as they reached the beach. (Paragraph 4)


The meaning of the underlined word is ….

A. Became out of order


B. Became full of creases
C. Failed to keep level with
D. Fell into very small pieces
E. Pushed a boat into the water.

The following text is for questions 27 and 28

Makassar: Fifteen people were injured after being hit by a car driven by 14-year-old
H.R.R. in Makassar, South Sulawesi, on Saturday.

The red Honda Jazz car, which belonged to H.R.R.’s father, was involved in a
sequence of five crashes on Jl. Baji Gau, Jl. Cendrawasih, Jl Dangko, at the Hartaco
housing complex and Jl. Dg Tata I, injuring 15 people, including three motorists and a
pedicab driver.

Angry residents damaged the car after the fifth crash, while H.R.R. was arrested by
the local police, tribunnews.com reported.

The accident reminded the public of a fatal accident in Jakarta on Jan. 22 in which
a highly intoxicated driver, Apriyani Susanti, slammed her car into pedestrians on Jl.
Ridwan Muis in Central Jakarta.

The accident killed eight people on the spot while the ninth victim died at Gatot
Subroto Army Hospital. Three other critically injured pedestrians were hospitalized.

27. What is the first information about?


A. The impact of the car hit in Makassar.
B. The condition of the victims of the car crash.
C. The anger of the residents about the car crash.
D. A lethal accident which occurred in South Sulawesi.
E. A teenager driver who hit many people in Makassar.

28. Which of these following spots is NOT the place where H.R.R. involved in car crash?
A. Jl. Cendrawasih.
B. Jl. Ridwan Muis.
C. Jl. Dg Tata I.

85
D. Jl. Baji Gau.
E. Jl Dangko.

The following text is for questions 29 and 30.


The Government should be more serious

Sometimes when I pass some certain places in Jakarta, I often see piles of trash by
the side of the street or even by the river. This view is annoying. It makes our city look ugly
and very dirty. If we walk near the pile of trash, we could smell stinky odors coming from it.

Piles of trash bring the bacteria with them. Isn’t that very dangerous to our health
if flies land on our food?

However, many people live around this mountain- like heap of trash. Many
children play nearby. Also, a lot of women wash clothes and fruits and vegetables in the
river which is contaminated by trash. This is terrible. I think they don’t realize the danger,
because they don’t have knowledge about healthy living.

In my opinion, the government must make a regulation that prohibits people to


build houses near the trash disposal (dump site). Also, there should be special officers,
like police officers, who watch and catch people who throw trash on the ground or into the
rivers. Moreover, if a big fine doesn’t make people stop dropping litter carelessly, the
government should put them in jail as punishment.

Finally, I think the efforts to keep our environment clean and tidy need hard work
from both the citizens and government.

29. What does the text tell us about?


A. How to live healthy.

B. The government’s problem.

C. The trash problem in Jakarta.

D. The police officers’ problem.

E. Children living around heap of trash.

30. The social function of the text is....


A. to share an amusing story with others

B. to inform the readers about environment problem

C. to present two points of view about environment

D. to describe the environment problem to the readers

E. to persuade the readers to be concerned with trash case

The following text is for questions 31 and 32.

Human body is actually a living machine. This living machine needs energy to
supply with. The energy is provided by the food, which we eat. However, do we know how
much food we need to stay healthy?

The energy value of food is usually measured in calories. A calorie is the amount
of heat, which is required to raise the temperature of 1 kg of water by 1 degree C. the
number of calories, which people need per day, varies. It depends on the activity, which
people are involved in. for example; people will need more calories for standing than for
sitting; people need more calories for running than for walking, etc.

86
The energy which is provided by food is in the form of three kinds of chemical
substances. They are carbohydrate, protein and fat. Carbohydrate provides 8.8 calories
per gram (cal/gm) of energy, protein 4.0 cal/gm and fat cal/gm. Each food contains
different proportion of these substances. These three chemical substances are important
for keeping body healthy.

31. What is the topic of the text?


A. Human body.
B. Energy for health.
C. Calorie measurement.
D. Chemical substances.
E. Important substances.

32. What is the measurement of food value?


A. Fat.
B. Heat.
C. Calorie.
D. Protein.
E. Carbohydrate.

The following text is for questions 33 and 34.

The Colosseum, or the Coliseum, originally the Flavian Amphitheatre (Latin:


Amphitheatrum Flavium, Italian Anfiteatro Flavio or Colosseo), is an elliptical amphitheatre
in the centre of the city of Rome, Italy, the largest ever built in the Roman Empire. It is
considered one of the greatest works of Roman architecture and Roman engineering.
Occupying a site just east of the Roman Forum, its construction started in 72 AD
under the emperor Vespasian and was completed in 80 AD under Titus, with further
modifications being made during Domitian's reign (81-96). The name "Amphitheatrum
Flavium" derives from both Vespasian's and Titus's family name (Flavius, from the gens
Flavia).
Capable of seating 50,000 spectators, the Colosseum was used for gladiatorial
contests and public spectacles such as mock sea battles, animal hunts, executions, re-
enactments of famous battles, and dramas based on Classical mythology. The building
ceased to be used for entertainment in the early medieval era. It was later reused for such
purposes as housing, workshops, quarters for a religious order, a fortress, a quarry and a
Christian shrine.
Although in the 21st century it stays partially ruined because of damage caused by
devastating earthquakes and stone-robbers, the Colosseum is an iconic symbol of
Imperial Rome. It is one of Rome's most popular tourist attractions and still has close
connections with the Roman Catholic Church, as each Good Friday the Pope leads a
torchlit "Way of the Cross" procession that starts in the area around the Colosseum.
(Taken from http://en.wikipedia.org/wiki/Colosseum-cite_note-wayofthecross-
frommers-5)

33. What is the text about?


A. Roman architectures.

B. Italian buildings.

C. The Colosseum.

D. The Gladiator.

E. The Battles.

87
34. When was the Colosseum completed?
A. In 72 AD.

B. In 80 AD.

C. In 21st century.

D. During Dominitian reign.

E. In emperor Vespasian’s era.

The following text is for questions 35 to 37.


Floods always happen in Indonesia. Floods begin when soil and vegetation cannot
absorb falling rain or when water runs off the land in such quantities that it cannot be
carried in normal stream channels or retained in natural ponds and human-made
reservoirs. Flash floods are the result of too much rain falling in too small area, in too short
time. Flash floods frequently occur in seconds and minutes, while floods occur over hours
and days.

Typically flash floods occur primarily at night and when there is an abundance of
atmosphere moisture; in addition, there is usually little, if any, vertical wind shear present.
Flash floods can be produced by large, slow-moving storms or as a result of “train effect”
storms.

Flash flood waves, moving at incredible speeds, can roll boulders, tear out trees,
destroy buildings and bridges, and scour out new channels. Damaging walls of water can
reach 10 - 20 feet. On small streams, water levels may rise quickly in heavy rainstorms,
flash floods are one of weather’s most dangerous effects.

35. What is the text mainly about?


A. The effects of flash floods.
B. The definition of flash flood.
C. The occurrence of flash floods
D. The ways to prevent flash floods.
E. The disadvantages of living in low land.
36. How does flash flood damage our environment?
A. It cannot roll boulders.
B. It moves at slow speed.
C. It goes through forest slowly.
D. It can destroy buildings and bridges.
E. It reaches less than 10 feet of walls of water.

37. What is the purpose of the text above?


A. To explain how flash floods happened.
B. To inform flash flood to the reader.
C. To describe flash flood.
D. To report flash flood.
E. To classify flash flood.

The following text is for questions 38 to 40.


Students’ cheating is one of the biggest problems faced by teachers nowadays.
Despite of the fact that teachers advise their students not to do dishonest acts during tests,
some students still do cheating. There are many kinds of cheating that students do during

88
the test or examination, such as looking at students’ work, put some notes under their
clothes, write notes or formulas on their desk, make hand signals or go to rest rooms to
get the answers from their friends.

In my view, students who do cheating are committing a crime. It is a sort taking


something illegally. This crime causes negative results to the students. Cheatings
deteriorate their mental capacity. Cheating acts also create dependence. They weaken
their self confidence. Instead of preparing their coming test, they are busy organizing notes
on a piece of paper. If this awful habit continues, the students will lose their opportunity to
develop their intellectual and mind.

Therefore, school should consider cheating as a very serious problem. School board
and administration should go hand to overcome this matter. Honesty must be put in school
vision. Harsh punishment must be applied to students who commit this crime.

38. What will the school do to eliminate students who commit cheating?
A. Create a good vision.
B. Select honest students.
C. Make harsh punishment.
D. Make students confident.
E. Administer the school affair.

39. What is mainly discussed in the text?


A. School policy.
B. Cheating problem.
C. Harsh punishment.
D. Mental deterioration.
E. Different ways of cheating.

40. What does paragraph one tell us about?


A. The students do not cheat during the test.
B. Students put some notes under their clothes.
C. The students write notes or formulas on their desk.
D. There are many kinds of cheating that students do during the test.
E. The students make hand signals or go to rest rooms to get the answers.

The following text is for questions 41 to 43.


Two students were discussing the school’s new rule that all students must wear a
cap and a tie. One of them showed her annoyance. She said that wearing a cap and a tie
was only suitable for a flag rising ceremony. Wearing a cap and a tie made the students
feel uncomfortable and hot. Moreover, the classrooms were not air-conditioned.
Contrary to the girl’s opinion, the other student was glad with it. He said that he did
not mind with the new rule because wearing a cap and a tie would make the students look
great and look real educated persons. He was sure that the students would wear them
proudly. They would surely be used to it any way.

41. What is the topic discussed?


A. The awareness for being discipline in time.
B. The obligation for students to wear a cap and a tie.
C. The benefit of wearing a cap and a tie for students.
D. The value added for the students to support their performance.
E. The reflection of students’ discipline towards to the school’s facilities.

89
42. The two students were discussing part of....
A. the facilities in school
B. their homework
C. their uniform
D. their friends
E. their family

43. The boy believed that all students would....


A. care for their environment
B. have a high spirit to study
C. solve their own problems
D. follow the new rule
E. feel uncomfortable

The following text is for questions 44 to 46.


After a terrifically enjoyable start, the Twilight series is settling into a somewhat
predictable groove, with its tragic-romantic motif of not getting married becoming a bit
gimmicky and worn. At the beginning of this film, directed by Chris Weitz, Bella (Kristen
Stewart) turns 18 and starts worrying about becoming the older woman to her eternally
youthful undead boyfriend Edward Cullen, played by Robert Pattinson. So, with his heart
audibly breaking, Edward finally bites the bullet - as it were - and leaves her for her own
good, settling apparently in Rio de Janeiro, that city being glimpsed subliminally just once.
Poor, post-breakup Bella mopes and whinges as the months drag past, taking refuge in
dangerous activities such as motorbike riding.

44. Who is the director of the film?


A. Bella Swan.
B. Chris Weitz.
C. Edward Cullen.
D. Kristen Steward.
E. Robert Pattinson.

45. What is the text mainly about?


A. A Recommendation to see the film.
B. Objection to distribute the film.
C. Distribution of the film.
D. Review on a film
E. Scenes of a film.

46. Why does Edward Cullen settle in Rio de Janeiro?


A. To get rid of Bella.
B. To form a new life.
C. To live peacefully.
D. To marry a woman.
E. To run a business.

Text for Question 47

The humphead wrasse is an enormous coral reef fish. It can grow over six feet long with a
prominent bulge on its forehead. Some of them live to be over 30 years old. They roam

90
through coral reefs in search of hard shelled prey such as molluscs, starfish, or
crustaceans.

WWF urges local governments in the Coral Triangle to stop the trade and consumption of
humphead wrasse—one of the most expensive live reef fishes in the world. Live reef fish
trade in Southeast Asia continues to be a significant problem that threatens the region’s
food security as well as its reefs, as poachers often resort to legal and destructive fishing
methods to catch them.

(Taken from https://www.worldwildlife.org/species/humphead-wrasse)


47. …… It can grow over six feet long with a prominent bulge on its forehead (P. 1). What
is the synonym of the underlined word?

A. Substantial
B. Noticeable
C. Important
D. Massive
E. Bold

Locals in the area of Copenhagen are familiar with bytestation or give box

Jakarta has been growing a lot and so has the lifestyle.

I went to private schools before high school. I didn’t understand then, but now I figured it out that
there were gaps between the haves and the have nots. I also didn’t really feel it in high school and
college, but it was still there.

There are more shopping malls with the latest fashions, more shops with big brands, more fancy
cafés and restaurants, etc – all things that young urbanites would love to spend their money (or
sometimes their parents’ money) on. Things and possessions become a priority, a way of life that
we will pass on to our children.

A heartwarming twist on the materialistic lifestyle is found in a cold country up north, Denmark.
Locals in the Verterbro area of Copenhagen are familiar with byttestation or give box. Bytte can
mean exchange in English, and as the name suggests, abyttesation is a box where everyone can
put in and take things out as an exchange.

In the byttestation, you can find things from baby clothes, shoes, cups and plates to furniture. This
byttestation is organized by local group called Naboskab (neighbourhood). Byttestations are also
common in cities like Berlin and Amsterdam, where they often go under the name of "give box" or
"swapping station."

The vision behind this project is to increase the share of direct recycled goods in the area. The
program encourages the perspective that it is fine to wear or use second hand goods. You can also
give away things you don’t like anymore that might be useful to somebody else. There’s no
obligation to how much the donation should be. You put things that you think can be useful to
others. It may be clothes, books, cookware and toys.

How can we make sure that nobody is taking advantage of the system? It’s a tricky question or
should I say it needs a tricky answer? It’s trust-based. We need to have self-discipline. We have to
respect others. This small thing can actually affect trust, security and the way to be “local” The
cooperation between local and social initiatives can be culture-creating for an entire area.

It is never wrong to channel your donations through foundations. But, maybe we could also restart
the idea with the genuine mind-set of helping. There are others who are not so lucky, so share and
care. Start with the little things around you.

Can we spread the idea of more sharing and less complaining?

91
(Summarized from http://www.thejakartapost.com/life/2017/10/03/byttestation-sharing-for-the-
community-and-the-earth.html)

48. We can learn from the text that the writer...


A. experienced the life in Copenhagen
B. understood the struggle of being poor
C. had good exprience with the byttestation
D. knew the characteristics of people in vertebro
E. understood the benefits of giving and sharing

I think my first trip to New Zealand was the best experience of my life. I met quate a
lot of nice people there. I made acquaintance with Jenny,Harry,Joseph, rexi, and Bryan.
They introduced me to their families. We had barbeque times in Rexi,s and Joseph,s
houses. Joseph and Jenny are brother and sister while Rexi and Bryan are twins.
You can find sheep almost everywhere in New Zealand. The tranquil meadow
where the sheep graze always appears in my mind every time. I go to sleep. Jenny,s
father has a lot of sheep and he sowed me his wool cutting machinery, which he uses
collect the sheep,s fur before processing it into thread.
For some people touring is a relaxing experience. Yet for me it completely the
apposite. I had to spend time more than 24 hours in the same clothes with greasy hair
and three hours of sleep. I would rather have traveled from Aucland to Milfort Sound and
more easy way . we could have spent a night or so in a hotel, not in our vehicle . However
, having a bunch or friend along the way perked me up. It make me want to come back to
New Zealand again.

49. What is the porpuse of the text?


a. To tell the readers about the writer’s trip’s to New Zealand.
b. The friendship of few teenagers
c. The situation in New Zealand.
d. The sheep in New Zealand.
e. The tiring holiday

50. According to the text, which of the sentences is NOT true?


A. The writer did not enjoy the touring.
B. She wants to visit New Zealand again.
C. She did not spend night in a hotel when touring.
D. The writer did not have barbeque in Bryan’s house
E. The writer tried collecting sheeps’ fur in New Zealand.

92
PREDIKSI SOAL UN TAHUN 2018

The last day of Elvis: The majestic musician's final battle with addictions

His obesity made it hard for him to walk and round-the-clock nursing staff had to help him
with everyday tasks as he struggled to waddle even a short distance.

His other addiction, to prescription drugs, only made things worse. In the first seven-and-a-
half months of 1977 alone, one doctor prescribed him more than 10,000 individual doses
of sedatives, amphetamines and narcotics and his daily cocktails of pills gave him a
slurring, zombie-like appearance.

“He was a mess”, his chief nurse Letetia Henley Kirk, who he fondly called Tish, recalls.

Today it is almost impossible to believe that – at the time – Elvis was still the most prolific
and highly paid live performer in America and had wrapped his latest tour two months
before on June 26 at Indiana Market Square Arena, Indianapolis.

Being on hiatus from touring and recording had seemed to accelerate his dreadful physical
descent and, according to his live-in girlfriend Ginger Alden, he had become more
depressed than ever and prone to “sudden outbursts of rage”.

To make matters worse on that August night, he had developed a severe toothache and
his dentist opened up his surgery in downtown Memphis for him. After the treatment, Elvis
and Alden returned to Graceland at midnight.

At first The King seemed to be in a better mood but within a couple of hours he
complained that his teeth “hurt like hell”, she recalls. At 2.15 am on August 16, Elvis woke
his personal doctor with a phone call for more pain medication, then woke his stepbrother
Rick Stanley to drive to the all-night pharmacy at nearby Baptist Memorial Hospital to pick
up six Dilaudid tablets.

At around 4am, feeling better and wide awake, Elvis played a game with his guest, after
one hour later, he decided to sleep and swallowed his small plastic bags of pills. At 7 am
he took a second pack of the twice-daily bags, possibly forgetting he had taken the first
only two hours earlier. However, at 9am, still unable to sleep, Elvis called down to
housekeeping to ask for a third bag, and at 9.30am he picked up the book he had been
reading – Frank Adams’s The Scientific Search For The Face Of Jesus – and told Alden,
by then awake, that he was heading for the bathroom. She recalls telling him: “Don’t fall
asleep in there,” knowing his habit of nodding-off while sitting on the toilet. “OK, I won’t,”
he replied. Alden went back to sleep, unaware that it was their final conversation.

Adapted from
http://www.express.co.uk/entertainment/music/839858/Elvis-Presley-death-anniversary-
drugs-addiction-obesity

93
16. What can we learn from the text?
A. Being a famous person, people need to consume some drugs
B. Elvis Presley dedicated his life for music around the world
C. Elvis had become more depressed than ever and prone to rage.
D. Being addicted to drugs, Elvis became losing his fame
E. The fame cannot guarantee someone’s life better
17. Why did The chief nurse think that Elvis was a mess? Because ….
A. Elvis Presley often made his nurse disappointed and angry
B. Elvis Presley was not a rock star anymore because of his obesity
C. Elvis Presley could not do his everyday task by himself
D. Elvis Presley used to ask his chief give him some packs of pill
E. Elvis Presley became overweight and addicted to drugs

VALDOSTA-LOWNDES COUNTY
Industrial Authority
June 1 2012
M. Johns Quarterman
3338 Country Club Rd L336
Valdosta, Georgia 31605

Dear Mr. Quarterman,

As you aware, the Valdosta-Lowndes County Industrial authority has recently


decided to move forward with a Target Business Analysis and is executing a contract with
Atlanta-based Market Street Service to facilitate this process. The Target Business
Analysis will be leveraged as the starting point for the holistic strategy and will be
supplemented by a comprehensive Competitive Assessment that expands on the
community profile findings.
During this process Market Street representatives will be conducting focus group
interviews with a number of community stakeholders. These stakeholders input sessions
will be conducted to inform both the Competitive Assessment and initial recommendations
to be included in the Economic Development Strategy phase.
We are inviting you to participate in a focus group interview on Wednesday, Jun
20th 4:30 pm-5:30 pm at Wiregrass Georgia Technical College, Lowndess Hall, Presidents
Board Room. It is very much hope you will agree to participate, lending your voice to the
discussion on economic development in the Valdosta region.
Please RSVP to Lu Williams or Donna Holland at 229-259-9972 by Monday, Jun
11th.
Thank you for considering this opportunity. We look forward to hearing from you.

Sincerely

Andrea Schruijer
Executive Director

94
18. Acording to the text, which ofthe following statement is correct?

A. The Valdosta Lowndese County invited Mr. Quarterman to be interviewed by the


board of this company
B. Mr. Quaterman was invited to do some business with Valdosta Lowndese County
C. Mr. Quaterman was a representative of Businessmen who was invited to join the
focus group interview
D. Mr. Quaterman represented the community stakeholders in attending the focus
group interview
E. Mr. Quaterman was one of representatives of the community stakeholders invited
to join Focus Group Interview

19. ....a comprehensive competitive assessment that expands on the community.... The
underlined word has the closest meaning to...

A. grow up
B. accelerate
C. increase
D. raise
E. fasten

Dear Rosa,

Hi Rosa, It has been long time no see you. I have a problem with my girlfriend. She is
twenty-five years old. We have been dating each other for years, since we were high
school students. Now we have jobs. I feel very sure that we love each other. Now it is
time for me to get married. Unfortunately, her parents never approve of our relationship
ever since the first time. My family, anyhow, does not really mind. At first I thought my
girlfriend had the power to defend our love. But then she surrendered. She loves me and
she loves her family, too. On the one hand she said, "I'll be happy if my family is happy." It
means she had to get approval from her parents.

For this I was shocked. My heart was broken. She ended our relationship just at the time
when I was ready to marry her. When I was with her, we were very happy. We had a lot in
common and there were no conflicts. The truth was, we separated only because of her
parents. They want her to marry her cousin, her aunt's son (this is one of the traditions in
the Karonese).

For the time being I am very upset. When I miss her, I call her. Then we can eat together,
talk together for hours. We act like elating because she loves me, too. The real problem
is, she cannot refuse what her parents want. So, should I forget my girlfriend, the only one
that I truly love? Or else, should I just wait, and dream that one day she will be back to
me?

Rio N., Padang

95
20. What is the relation between Rosa and Rio?
A. They are employers.
B. They are pen pals.
C. They are classmates
D. They are roommates
E. They are relatives

21. Why do the girl's parents disapprove of their marriage? Because ….


A. they don't like Rio
B. the girl is still studying
C. Rio is from a different ethnic group
D. they keep the Karonese tradition
E. the girl has chosen another guy

22. We act like elating because she loves me, too. (The last paragraph, line 2). What is the
closest meaning of the underlined word?
A. thrilling
B. flying
C. lifting
D. developing.
E. trembling

You must have been aware of the growing need to conserve our forests. One of
the ways to ensure that fewer trees are cut down is by recycling waste paper. This is the
reason why our Conservation Club has embarked on our Old Newspaper Collection
Campaign.
Perhaps you would be interested in knowing what will happen to your old
newspaper after they arrive at the paper recycling center.
Firstly, the old newspapers are soaked in water and then put into hydropulper.
This machine, which has rotating blades, cuts the paper finely into a pulp.
Next, the pulp is fed into the washer. Here, all dirt is washed off. Staples and clips
are removed. The pulp is then fed into the chemical tank where bleach is added to it. This
whitens the pulp.
After being bleached, the pulp is passed through rollers. All excess water is now
squeezed out the pulp. This makes the drying process faster. The drying is done in a
dryer. The pulp comes out of the dryer as large sheets of paper, which are rolled on
spools.
This recycled paper is used for many things, for example, to make cardboard
boxes, egg cartons, toilet rolls, poster paper, and even gift wrapping paper. When you
consider that you have no further use for your old newspaper, you will be glad to contribute
them to our campaign.

96
23. The text tells us about ....
A. the process of recycling old newspapers
B. the need to conserve our forests
C. a conversation club
D. an old newspaper collection campaign
E. the efforts to stop cutting down trees in forests

24. The old newspaper is put into hydropulper after ….

A. the pulp is fed into the washer.


B. all dirt is washed off.
C. it is soaked in water.
D. Staples and clips are removed.
E. The pulp is then fed into the chemical tank

How to make a French toast

Recipe for French Toast

You are going to need:

4 pieces of bread
1 spoon of sugar
2 eggs
A quarter of a cup of milk
Butter
Pan
Fork
Bowl

Before you start to cook, you have to read the recipe.


Now you can get ready. After you read the recipe, put everything on the counter.
When everything is ready, break the eggs, pour a quarter of the milk in the bowl, then add
a small spoon of sugar. Mix the eggs, milk and sugar.

Next, put a piece of bread in the bowl with the eggs, milk and sugar. Turn over the bread.

Now, put some butter in the pan. Turn on the stove. When the pan is hot, take the bread
out of the bowl and put it into the pan. After you cook one side of the bread, cook the other
side. After you finish the first piece of the bread, cook the other pieces. Now you have
French toast!

97
25. The purpose of the text is …
A. to describe French toast
B. to explain about French toast
C. to entertain the reader with French toast
D. to tell the reader how to make French toast
E. to persuade the reader to make French toast

26. Who is the text better addressed to?


A. An expert chef
B. A mother at home
C. A student learning at home
D. A husband left by his wife
E. A boy/girl left by his /her mother

Polar Bear

In spring 2008, the polar bear was placed on the endangered species list. According to
the USA's Endangered Species Act, an endangered species is an animal, plant or any
other kind of wildlife that is likely to face extinction in its natural habitat. Polar bears had
already been categorized as a "threatened" species which the ESA defines as one that is
likely to become "endangered" in the foreseeable future.

The polar bear is the first animal that has been classified as endangered due primarily
to global warming. Global warming is a form of climate change caused by increased
levels of carbon dioxide and other greenhouse gases that become trapped in the
atmosphere. Oil and gas heaters, engines using oil or petrol (or gasoline) and coal-
powered electricity plants all require fossil fuels that lead to global warming. Renewable
energy sources or renewables that don't cause global warming include solar, wind and
hydro-electricity power. These alternative forms of power, as well as the science of climate
change, have been strongly attacked by many of the world's biggest companies in order to
protect the huge profits they make from fossil fuels. They do this even though they're fully
aware of how much fossil fuels are damaging our planet and endangering many of the
species we share it with.

The polar bear's habitat is more vulnerable to global warming than many other species.
Polar bears live mainly on the sea ice in the Arctic. This is where they hunt for fish and
build up fat reserves. When the ice melts many polar bears move to land and live off their
stored fat. In the Arctic, global warming is causing the ice to melt slightly earlier and form
slightly later. This results in a shorter feeding season for the polar bear. Some risk their
lives to find ice. If they have to swim too far they will drown from exhaustion and hunger.
The World Wildlife Fund estimates that over 25% of Arctic sea ice has disappeared in the
past 30 years.

It is not only the polar bear that is at risk in the Arctic. Every species of plant, animal, and
insect there is threatened by global warming. Nevertheless, groups of concerned scientists

98
and environmentalists such as the World Wildlife Fund often study large carnivores in
order to assess the health of an ecosystem. The Arctic food chain relies on the polar
bear. In addition, donations are more commonly offered for the protection of large animals
such as bears or elephants. People in general are less interested in conservation
efforts that protect small wildlife, such as plants or insects. However, by using donor
money to protect the habitats of larger animals through reforestation and similar
programs, entire ecosystems with many threatened or endangered species can be saved.

27. What is the purpose of the text?

A. To inform that Polar Bears become endangered animals


B. To inform that the habitat of Polar Bear become extinct
C. To persuade the readers to save Polar Bear
D. To persuade the readers to give charity to Polar Bear
E. To inform that global warming is dangerous for animals

28. What major change has occurred in the polar bear's natural habitat?

A. Colder temperature
B. Uncontrolled exploration in arctic
C. Severed exhaustion and hunger
D. Greenhouses gases effect
E. Disappearing ice sea

29. We can conclude that ….

A. the polar bear has been facing the extinction because of the growth of human
population
B. the global warming causes only the habitat of Polar bear disappeared rapidly
C. the global warming causes the sea ice of Arctic disappeared
D. the global warming has been threatening every creatures in the world
E. to protect the ecosystem, it is needed the awareness of all people around the world

30. .... and live off their stored fat. Which sentence has the closest meaning to the
underlined word?

A. They stay for awhile


B. My father grows some flowers
C. My father earns money for us
D. The tiger lives for hunting
E. The insect is threatened for living

99
What is the benefit of tourism for local people? Well, Tourism is now a huge
contributor to the economies of most countries. Tourism industries can bring money, job
vacancy and advancement especially to developing regions. However, this money often
goes into the pockets of foreign investors, and only rarely benefits for local people.
Tourism industries will not give much benefit for local people if, for example,
multinational hotel chains don't care about the surrounding nature when they build new
hotels. This can cause many social, cultural and geographical problems. Some local
people may get job and money from that International hotel chain. However in case of
missing that opportunity, some of them still have their own environment.
Moreover, some facts show that tourists tend to go, visit and spend their money in
restaurants, bars and even luxury hotels of that multinational chains. They less go to such
places; restaurant, bar, hotel, shop which are owned by local people. This can prevent the
local people's business from becoming even larger.
Most important thing, tours or excursions of tourism have little effect on nature.
Even it can disrupt or destroy ecosystems and environments, and if it does, the local
people will get the risk.
So the local government policies should be put in place to ensure that tourism will
make the benefit spreading widely. The policies should guarantee that tourism will not
cause any harm to any local people or places.

31. What issue which makes the writer aware?


A. The multinational chains prevent the local people’s business to become larger.
B. The profit of tourism often goes to foreign investor, not to local people
C. Tourism gives a huge contributor to the country
D. Tourism gives a big job opportunity
E. Local people earn much money from tourism business

32. What is the writer’s opinion about the issue?


A. Luxury hotels are commonly owned by local people
B. Excursions of tourism have no effect on nature.
C. The policy should guarantee that tourism will not give benefit to local people.
D. Tourists tend to go, visit and spend their money in restaurants, bars.
E. Government’s policy tends to be beneficial for local people.

33. From the text we know that


A. Tourism will never benefit local people.
B. The government policies improves the welfare of local people.
C. Tourists tend to go, visit and shop in restaurant and shop which are owned by local
people
D. Local people cannot get job and money from that International hotel chain.
E. Tourism is now a huge contributor to the economies of most countries.

100
34. Tourism industries will not give much benefit for local people … the multinational
hotel do not pay attention to the surrounding nature.
A. so
B. as
C. for
D. if
E. and

Indian nationalist leader Mahatma Gandhi (born Mohandas Karamchand


Gandhi) was born on October 2, 1869, in Porbandar, Kathiawar, India, which was then part
of the British Empire. Mahatma Gandhi’s father, Karamchand Gandhi, served as a chief
minister in Porbandar and other states in western India. His mother, Putlibai, was a deeply
religious woman who fasted regularly.
At the age of 13, Mahatma Gandhi wed Kasturba Makanji, a merchant’s
daughter, in an arranged marriage. In 1885, he endured the passing of his father and
shortly after that the death of his young baby. In 1888, Gandhi’s wife gave birth to the first
of four surviving sons. A second son was born in India 1893; Kasturba would give birth to
two more sons while living in South Africa, one in 1897 and one in 1900.
Young Gandhi was a shy, unremarkable student who was so timid that he slept
with the lights on even as a teenager. In the ensuing years, the teenager rebelled by
smoking, eating meat and stealing change from household servants.
Although Gandhi was interested in becoming a doctor, his father had hoped he
would also become a government minister, so his family steered him to enter the legal
profession. In 1888, 18-year-old Gandhi sailed for London, England, to study law. The
young Indian struggled with the transition to Western culture.
Upon returning to India in 1891, Gandhi learned that his mother had died just
weeks earlier. He struggled to gain his footing as a lawyer. In his first courtroom case, a
nervous Gandhi blanked when the time came to cross-examine a witness. He immediately
fled the courtroom after reimbursing his client for his legal fees.
After struggling to find work as a lawyer in India, Gandhi obtained a one-year
contract to perform legal services in South Africa. In April 1893, he sailed for Durban in the
South African state of Natal.

35. Which of the following information clearly describes about Gandhi?


A. Mahatma Gandhi could save his first baby’ life.
B. Gandhi’s family support him to become a doctor.
C. Mahatma Gandhi was born in Kathiawar in 1896.
D. Mahatma Gandhi got married before 13 years of age.
E. Mahatma Gandhi spent 3 years learning laws in England.

36. We learn from the text that ….


A. Gandhi lost his first child after his mother’s death.
B. When Gandhi was young he was a brave man.
C. Gandhi’s parents let him decide his program for his study.

101
D. Gandhi felt so easy to find his job as lawyer in India.
E. Gandhi kept his own culture when he lived in London

37. Mahatma Gandhi experienced a rebelling life when ….


A. Gandhi lived in London and refused smoking.
B. Gandhi broke the rules as Hinduism follower.
C. Gandhi found himself as a young brave man.
D. Gandhi felt so easy to find his job as lawyer in India.
E. Gandhi kept his own culture when he lived in London

38. He immediately fled the courtroom after reimbursing his client for his legal fees.
The underlined word means ….
A. got into
B. went out
C. ran away
D. rush
E. escape

It is commonly observed during the rainy season that water falling on roads and
streets disappears after a few hours. Similarly, in summer, wet clothes dry up very soon.
Do you know where this water goes? This water gets converted into vapor called
evaporation.

How does evaporation take place? Every substance is made up of very small particles
called molecules. These molecules are held together by strong forces of attrition called
cohesive forces. These forces are opposed by the repulsive forces due to the motion of
molecules. As long as the cohesive forces are far greater than the repulsive forces, the
substance remains in the solid state. When the substance is heated up, it absorbs heat
energy which the molecules area set up in rapid motion. This motion starts
counterbalancing the cohesive force. When the repulsive force generated by the motion of
molecules equals the cohesive force, matter changes from solid to liquid state. If the liquid
is still heated, the molecules still move fast. When the force exceeds the cohesive force,
the molecules of the liquid become free and escape into the air. It is how the liquid is
converted into vapor. The liberation of the molecules from the liquid surface to the air is
called evaporation.

This explains the drying up process of clothes. If the wet clothes are put under the sun,
they dry up faster because at the rate of evaporation increases with the rise in
temperature. It takes place more rapidly when the air is dry. That is why clothes dry up
more quickly on a dry than on a damp cloudy day.

102
39. What is the text about?
A. Do you know where this water goes?
B. How does evaporation take place?
C. How liquid is converted into vapor.
D. This water gets converted into vapor.
E. This explains the drying up process of the clothes

40. Evaporation usually happens when the molecules ….


A. move fast.
B. escape into the air.
C. become free
D. set up in rapid motion
E. release from the liquid float up to the air.

103

Вам также может понравиться